Sei sulla pagina 1di 76

REGULATORY FRAMEWORK FOR BUSINESS TRANSACTIONS

FIFTH YEAR
____ 1.
I A director is removed from office by a vote of the stockholders holding or
representing at least 2/3 of outstanding capital stock. The vacancy occasioned by
such removal can be filled by the vote of a majority of the remaining directors if
still constituting a quorum.
II The vacancy in the board occasioned by the death of a director can be filled by the
vote of a majority of the remaining directors if still constituting a quorum.

a. False, false
b. False, true
c. True, false
d. True, true
____ 2.
I A director is removed from office by a vote of the stockholders holding or representing
at least 2/3 of outstanding capital stock. The vacancy occasioned by such removal can
be filled by the vote of at least a majority of the remaining directors if still constituting
a quorum.
II Treasury shares sold for less than par or issued value are considered “watered stock”
and as such are prohibited by law.

a. True, true
b. True, false
c. False, false
d. False, true

____ 3. A foreign company has been exporting goods to a Philippine company for several years now.
When the Philippine company failed to pay the latest exportation, the foreign company sued to collect in the
Philippines. The Philippine company interposed the defense that the foreign company was doing business in
the Philippines without a license; hence, could not sue before a Philippine court. Is this defense tenable?
a. The defense is not tenable. A foreign corporation may sue in the Philippines whether or
not they have license to do business in the Philippines.
b. The defense is tenable because the foreign corporation is already guilty of estoppel.
c. The defense is not tenable.The mere act of exporting from one’s own country, without
doing any specific commercial act within the territory of the importing country. Thus, the
foreign company may sue in the Philippines despite lack of license to do business in the
Philippines.
d. The defense is tenable, foreign corporation without license to do business cannot sue in the
Philippines.

____ 4.
I If the surplus profits of the stock corporation reaches the level equal to its paid-up
capital, the SEC may compel the corporation to declare dividends, otherwise it will be
liable for a surtax on improperly accumulated surplus.
II In a corporation, two or more positions may be held concurrently by the same person
except that no one person shall act as President and Chairman of the Board.

a. False, false
b. True, false
c. False, true
d. True, true

____ 5.
A statement in writing by the pledgee that he renounces or abandons the pledge is sufficient
to extinguish the pledge only if accepted by the pledgor or owner and the thing pledged is
returned.
At the public auction, the pledgee may also bid and his offer shall be valid even if he is the
only bidder.

a. Both are false

Page 1 of 76
b. Only the first is true
c. Only the second is true
d. Both are true

____ 6. A clause providing that the mortgagee will automatically own the property mortgaged if the
debt is not paid at maturity is
a. Pactum commisorium
b. Payment by cession
c. Dacion en pago
d. Upset price

____ 7. A, B and C are partners. D is admitted as a new partner. Will D be liable for partnership
obligations contracted prior to his admission to the partnership?
a. Yes, but his liability will extend only to his share in the partnership property and not to his
own individual property
b. No, only for those contracted after his admission
c. Yes, and his liability would extend to his own individual property
d. Yes, as if he had been an original partner
____ 8. Which of the following expresses a correct principle of law. Choose the best answer.
a. A threat to enforce one’s claim through competent authority, if the claim is legal or just,
does not vitiate consent.
b. Simulation of a contract always results in a void contract.
c. Violence or intimidation does not render a contract annullable if employed not by
contracting party but by a third person.
d. Failure to disclose facts when there is a duty to reveal them, does not constitute fraud.
____ 9. In negotiorum gestio, the officious manager shall be liable for fortuitous event under the
following except?
a. He takes charge of abandoned property without the consent of the owner.
b. He undertakes risky operations which the owner was not accustomed to embark upon.
c. He fails to return the property upon demand by the owner.
d. He assumed the management in bad faith.

____ 10. Knowing that the car had a hidden crack in the engine, X sold it to Y without informing the
latter about it. In any event, the deed of sale expressly stipulated that X was not liable for hidden defects.
Does Y have the right to demand from X a reimbursement of what he spent to repair the engine plus
damages?
a. No, because Y is in estoppel, having changed engine without prior demand.
b. No, because Y waived the warranty against hidden defects.
c. Yes, X is liable whether or not he was aware of the hidden defect.
d. Yes, since the waiver is void. X knew of it but he acted in bad faith in not disclosing the
fact to Y.
____ 11. A sold to B a fake Rolex watch on January 3, 2015. On January 13, 2015, B discovered that
the watch he bought from A was an imitation. The law provides that he can annul the sale as avoidable
contract within four years. Prescription starts from:
a. The time of delivery of the watch to B.
b. January 3, 2015 when the sale was perfected.
c. January 13, 2015 when the fraud was discovered by B.
d. The time they first talked about the sale of the watch.
____ 12. X, a customer, drinks a bottle of soft drinks in a restaurant and is hospitalized due to harmful
substance in the beverage. Is the manufacturer liable for damages in the absence of contractual relation
between them?
a. Yes, because its liability arises out of quasi-contract.
b. Yes, because of quasi delict.
c. No, there being no contract from which liability may arise.
d. Yes, because there is actually a contract between them, the restaurant being the agent of
the manufacturer.

____ 13. At the annual meeting of ABC Corporation for the election of five directors, A, B, C, D, E, F
and G were nominated. A, B, C, D and E received the highest number of votes and proclaimed elected. F
received ten votes less than E. Subsequently, E sold his shares to F, who between E and F has the right to
attend as director in the board meeting? The transfer of shares having been registered with the corporation.
a. E is the director because his term is one year until his successor is elected and qualified.
b. F is the director for he has acquired all the shares of E.

Page 2 of 76
c. Neither of them shall be the director
d. Either of them shall be the director.

____ 14. X, who was abroad, phoned his brother, Y, authorizing him to sell X’s parcel of land in
Pasay. X sent the title to Y by courier service. Acting for his brother, Y executed a notarized deed of absolute
sale of the land to Z after receiving payment. What is the status of the sale?
a. Valid, since a notarized deed of absolute sale covered the transaction and full payment was
made.
b. Valid, since the buyer could file in action to compel X to execute a deed of sale.
c. Void, since X should have authorized agent Y in writing to sell the land.
d. Valid, since Y was truly his brother X’s agent and entrusted with the title needed to effect
the sale.
____ 15. Under this doctrine, the corporation may exercise only powers expressly authorized by law or
incident to its existence:
a. Doctrine of separate juridical personality
b. Doctrine of limited capacity.
c. Trust fund doctrine
d. Doctrine of piercing the veil of corporate entity

____ 16. A, B and C are co-owners of a parcel of land. A sold his 1/3 share on April 7, 2016, B sold
his 1/3 share on April 15, 2016 and C his 1/3 share on April 23, 2016, all to D with the right to repurchase.
Which of the following is correct?
a. A may redeem his 1/3 share only if D allows him to do so.
b. A can redeem his 1/3 share even if D requires him to redeem the whole property.
c. A should redeem the whole property if he is required by D to do so.
d. A can redeem the entire property even if D allows him to redeem only his 1/3 share
____ 17. When the debtor binds himself to pay when his means will permit him to do so, the obligation
is:
a. pure
b. conditional
c. simple
d. with a period
____ 18. Sometimes referred to as condition precedent
a. Potestative condition
b. Casual condition
c. Suspensive condition
d. Resolutory condition

____ 19. The draft made in the Philippines calls for payment in Canadian dollars.
a. The draft is nonnegotiable because it calls for payment in money of another country.
b. The instrument is negotiable if it satisfies all of the other elements of negotiability.
c. The instrument is negotiable only has the exchange written on the draft.
d. The draft is nonnegotiable because the rate of exchange may fluctuate thus violating the
sum certain rule.
____ 20.
I Loss of a generic thing which is the object of an obligation, even without the fault of
the debtor does not extinguish the debtor’s obligation.
II An example of an obligation with a period is when Gary promises to pay Martin P1
million one month from the death of Janno.

a. False, false
b. True, false
c. False, true
d. True, true
____ 21. A, B and C are solidary debtors of W and Y, joint creditors for P12,000 where the share of
the debtors in the obligation is 2:3:5 while the share of the creditors is 1:2. If A is insolvent, which of the
following is correct?
a. W or Y can collect from B and C P12,000
b. Y can collect from C P6,400
c. Y can collect from either B or C P8,000
d. W can collect from B P3, 200

Page 3 of 76
____ 22.
I A donated to B a real property, the donation is made orally.
II A sold to B a real property, the sale is made orally

a. Both contracts are void.


b. Only the second contract is valid
c. Both contracts are valid
d. Only the first contract is valid.
____ 23. The following contract must be in a public instrument for the purpose of convenient. Which
is the exception?
a. cession of conjugal partnership of gain
b. repudiation of hereditary right
c. donation of real property
d. power to administer property
____ 24. This is the deposit of the thing due made by the debtor in lawful form, whenever the creditor
refuses or cannot accept payment.
a. deposit
b. consignation
c. mutuum
d. pledge

____ 25. Which of the following is not correct about partner’s right over specific partnership property?
a. A partner’s right in specific partnership property is not assignable.
b. A partner is co-owner with his partners of specific partnership property.
c. A partner’s right in specific partnership property is not subject to legal support.
d. A partner’s right in specific partnership property is not subject to attachment or execution
even on a claim against the partnership
____ 26. X bought a land from Y, paying him cash. Since they were friends, they did not execute any
document of sale. The heirs of X asked after Y to execute a deed of absolute sale to formalize the verbal sale
to their father. Unwilling to do so, X’s heirs filed an action for specific performance against Y. Will their
action prosper?
a. No, since the sale cannot under the Statute of Frauds to be enforced.
b. Yes, after full payment, the action became imprescriptible.
c. No, the action to enforce the verbal agreement has already elapsed.
d. Yes, since X bought the land and paid Y for it.
____ 27. If one party was mistaken and the other acted fraudulently or inequitably in such a way that the
instrument does not show their true intention, which one of the following is correct?
a. The former may ask for annulment
b. Either the former or the latter may ask for reformation
c. The latter may ask for reformation
d. The former may ask for reformation
____ 28. A bought A parcel of land from B on installment. When the first installment fell due, A did
not pay. His defense was that he did not have available money, And he therefore pleaded impossibility of
performance.
a. A is liable because the obligation is to pay specific money.
b. A obligation is extinguished because A has no available money.
c. A is liable. A mere pecuniary inability to fulfill An engagement does not discharge the
obligation of the contract.
d. A obligation is extinguished because of impossibility of performance.
____ 29. In 2018, L, M, N, O and P formed a partnership. L, M and N were capitalist partners who
contributed P500,000 each, while ), a limited partner contributed P1,000,000. P joined as an industrial partner,
contributing only his services. The Articles of Partnership, registered with the SEC, designated L and O as
managing partners; L was liable only to the extent of his capital contribution; and P was not liable for losses.

In 2019, the partnership earned a net profit of P800,000. In the same year, P engaged in a different business
with the consent of all the partners. However in 2007, the partnership incurred a net loss of P500,000. In
2008, the partners dissolved the partnership. The proceeds of the sale of partnership assets were insufficient to
settle its obligation. After liquidation, the partnership had an unpaid liability of P300,000.

Can the partnership creditors hold L, O and P liable after all the assets of the partnership are exhausted?

Page 4 of 76
a. No. P is not liable because there is a valid stipulation exempting him from losses. Since
the other partners allowed him to engage in an outside business activity, the stipulation
absolving P from liability is valid. For O, it is basic that a limited partner is liable only up
to the extent of his capital contribution.
b. Yes. The stipulation exempting P from losses is valid only among the partners. L is liable
because the agreement limiting his liability to his capital contribution is not valid insofar
as the creditors are concerned. Having taken part in the management of the partnership, O
is liable as capitalist partner.
c. Yes. The stipulations exempting P and L from losses are not binding upon the creditors. O
is likewise liable because the partnership was not formed in accordance with the
requirements of a limited partnership.
d. No. The Civil Code allows the partners to stipulate that a partner shall not be liable for
losses. The registration of the Articles of Partnership embodying such stipulations serves
as constructive notice to the partnership creditors.

____ 30. Choose the situation which illustrates best the minimum requirement of the law to corporate
formation:
a. Authorized capital (50,000); Subscribed capital (12,500); Paid-in capital (3,125)
b. Authorized capital (150,000); Subscribed capital (37,500); Paid-in capital (15,000)
c. Authorized capital (60,000); Subscribed capital (30,000); Paid-in capital (5,000)
d. Authorized capital (600,000); Subscribed capital (150,000); Paid-in capital (37,500)
____ 31. X subscribed 10,000 shares in the capital stocks of AAA Corporation. He paid 50% of the
10,000 shares. X asked the Corporate Secretary to issue him the corresponding stock certificate representing
the 50% of what he already paid. The Corporate Secretary of the corporation refused. Was the Corporate
Secretary correct?
a. The Corporate Secretary cannot refuse because a Stock Certificate can be issued
corresponding to the percentage of shares which were paid.
b. The Corporate Secretary is correct because the Corporation Code provides that no
certificate of stock shall be issued to a subscriber until the shares as subscribed have been
fully paid.
c. The Corporate Secretary cannot refuse because it is his legal duty to issue a stock
certificate corresponding to the number of shares actually subscribed regardless of the
actual payment.
d. The Corporate Secretary cannot refuse because a Certificate of Stock can be issued
provided it is indicated in the Certificate the actual percentage of what has been paid.

____ 32. A is indebted to solidary creditors W, Y and Z for P90,000 due on August 1, 2016, while W
owed A P90,000 also due on August 1, 2016. On August 1, 2016
a. Both obligations are extinguished by compensation
b. Either Y or Z can demand P90,000 from W.
c. Either Y or Z can demand P60,000 from A.
d. A’s obligation is extinguished up to P30,000
____ 33. A and B put up a partnership to engage in distribution of books and school supplies. A
contributed P1M while B his services. A wants to put up a restaurant on the opposite side of the street. On the
other hand, B wants to have a bakery beside A’s restaurant. Which of the following is correct?
a. A may put up his restaurant without need of securing B’s consent
b. B may put up his bakery without need of securing A’s consent
c. Both A and B can put up their restaurant and bakery businesses without the need of securing
each other’s consent
d. Neither A nor B can put up another business
____ 34. P orally appointed A as his agent to sell the former’s land. On January 3, 2015, A sold the
land to B who forthwith took possession thereof. It turned out however, that on January 1, 2015, P, without
informing A, had already sold the same land to C, who up to now has not taken possession of the same land.
Neither of the sales was registered. Whose contract shall prevail?
a. The sale to B for the agent was duly authorized to sell the land.
b. The sale to B for he was first possession in good faith.
c. The sale to C for the land was first sold to him by the owner.
d. The sale to C because the sale to B was void. A was not duly authorized by P.

____ 35. One of the requirements needed for a holder of a negotiable instrument to be a holder in due
course is the value requirement. Rodrigo is a holder of a P10,000 check written out to him. Which of the
following would not satisfy the value requirement?

Page 5 of 76
a. Rodrigo received the check in exchange for a promise to do certain specified services
three months later.
b. Rodrigo took the check in exchange for a negotiable note for P11,200 which was due on
that day.
c. Rodrigo received the check for a tax service debt for a close relative.
d. Rodrigo received the check from a tax client to pay off a four-month-old debt.
____ 36. In which of these cases, is advance payment recoverable by the debtor?
a. if the debtor believed that the obligation was already due and demandable.
b. if the debtor was not unaware of the period.
c. if the payment is only of interest credited for the proper period.
d. if the obligation is reciprocal and both parties advanced payment.
____ 37.
I Directors as a rule are entitled to compensation as directors but not exceeding ten
percent of the net income before tax of the preceding year.
II All private corporations in the Philippines shall be incorporated under the Corporation
Code which is a general law.

a. True, false
b. False, true
c. True, true
d. False, false
____ 38. M and N were good friends. N borrowed P1 million from M. Because of their close
relationship, the promissory note executed by N provided that he could pay the loan “whenever his means
permit”. Subsequently, M and N quarreled. M is now demanding immediate payment from N to which the
latter refuses. Decide.
a. It is an obligation with a suspensive condition dependent upon the debtor’s will and
therefore is void.
b. It is an obligation which is demandable at once because no period was agreed upon by the
parties.
c. It is an obligation with a period which depends upon the debtor’s will which the court may
fix.
d. The obligation is one with a condition dependent upon the capability of N to pay who shall
be obliged to pay only if he has the money.
____ 39.
If property has been promised by a partner as contribution to the partnership, the fruits arising from
the time the property should have been delivered should also be given provided prior demand was
made.
A partner who has undertaken to contribute a sum of money and fails to do so becomes a debtor for
the interest and damages from the time he should have complied with his obligation, without the
need of any demand.

a. Both statements are true


b. Only the second statement is true
c. Both statements are false
d. Only the first statement is true
____ 40. Legal compensation is allowed when one of the debts
a. consists in civil liability arising from a penal offense
b. arises from a claim for support by gratuitous title
c. arises from a sale of real property made by a minor to a capacitated person.
d. arises from the obligation of a depositary

Answer key:
B
C
C
A
A
A

Page 6 of 76
A
A
A
SOL:Negotiorum gestio is the ‘unauthorized administration’. Roman law gave an action, the direct
action, against a negotiorum gestor, a person who involved himself in the affairs of another without
instruction. There was, however, no action if the gestor acted with the care he used in his own
affairs, and, indeed, the gestor had a contrary action for the expenses involved in doing the work.
The action, to be found in many civilian jurisdictions, has become the foundation for a wider law of
restitution, France being an example. It is known in Scotland but is wholly rejected in England,
although the rapidly developing law of restitution there may result in some of these issues being re-
examined.
D
C
B
C
C
B
B
D
C
B
D
C
B
C
B
D
D
D
C
B
D
B
A
A
D
A
A
B
C
B
C

Page 7 of 76
ADVANCED FINANCIAL ACCOUNTING AND REPORTING
FIFTH YEAR

____ 1. The following selected accounts appeared in the trial balance of United Sales as of December
31, 2020

Debit Credit
Installment Receivable - 2019 sales 15,000
Installment Receivable - 2020 sales 200,000
Inventory, December 31, 2019 70,000
Purchases 555,000
Repossession 3,000
Installment Sales 425,000
Sales (regular) 385,000
Unrealized gross profit - 2019 54,000

Additional Information:
Installment receivable - 2019 sales, as of December 31, 2019, P120,000.
Inventory of new and repossessed merchandise as of December 31, 2019, P95,000
Gross profit percentage of regular sales during the year, 30% on sales.
Repossession was made during the year. It was a 2019 sale and the corresponding
uncollected account at the time of repossession was P7,750

The gain (loss) on repossession in 2020:


a. 1,262.50
b. (1,262.50)
c. 1,805
d. (1,805)
____ 2. Kemi, a private limited company, has acquired 100% of Coal, a private limited company, on
January 1, 2012. The fair value of the purchases consideration was 10 million ordinary shares of P1 of Kemi,
and the fair value of the net assets acquired was P7 million. At the time of the acquisition, the value of the
ordinary shares of Kemi and the net assets of Coal were only provisionally determined. The value of the
shares of Kemi (P11 million) and the net assets of Coal (P7.5 million) on January 1, 2012, were finally
determined on November 30, 2012. However, the directors of Kemi have seen the value of the company
decline since January 1, 2012 and as of February 1, 2013 , wish to change the value of the purchase
consideration to P9 million. What value should be placed on the purchase consideration and assets of Coal as
at the date of acquisition?
a. Purchase consideration P10 million, net asset value P7 million
b. Purchase consideration P11 million, net asset value P7.5 million
c. Purchase consideration P9 million, net asset value P7.5 million
d. Purchase consideration P11 million, net asset value P 7 million.
____ 3. Morris, Inc. acquires 100% of the voting stock of Crystal Company on January 1, 2019 for
P400,000 cash. A contingent payment of P16,500 will be paid on April 15, 2020 if Crystal generates cash
flows from operations of P27,000 or more in the next year. Morris estimates that there is a 20% probability
that Crystal will generate at least P27,000 next year and uses an interest rate of 5% to incorporate the time
value of money. The fair value of P16,500 at 5% having a probability weighted approach is P3,142.

Assuming Crystal generates cash flow from operations of P27,200 in 2019, how will Morris record the
P16,500 payment of cash on April 15, 2020?
a. Debit: Contingent performance obligation (P16,500) and Credit: Cash (P16,500)
b. Debit: Contingent performance obligation (P3,142); Debit: loss from contingent
performance obligation (P13,358) and Credit: Cash (P16,500)
c. Debit: Investment in Subsidiary and Credit: Cash (P16,500)
d. Debit: Goodwill and Credit: Cash (P16,000)
____ 4. Following is the balance of the ABCD Partnership at March 31, 2013 when the partnership is
to be liquidated:

Cash 6,000 Liabilities 12,400


Other assets 126,000 A, loan 12,000
B, loan 14,400
D, loan 9,600

Page 8 of 76
A, capital -25% 16,200
B, capital -25% 12,000
C, capital -25% 37,700
D, capital -25% 17,700

During the month of April 2013, assets having a book value of P18,000 are sold at a loss of 2,400.
Liquidation expenses of P600 are paid as well as P7,200 of the liabilities. Of the liabilities shown in the
balance sheet, P240 represents salary payable to D and P160 represents salary payable to C.

On April 30, 2013 cash to be distributed to A, B, C and D as follows:


a. A (0); B (0); C (0); D (9,000)
b. A (1,950); B (1,950); C (1,950); D (1,950)
c. A (0); B (0); C (0); D (1,950)
d. A (0); B (0); C (9,000); D (0)
____ 5. From the following data from the records of ABC partnership:

Cash 2,000
Other Noncash Assets 28,000
Total assets 30,000

Liabilities 5,000
A, loan 2,500
A, capital 12,500
B, capital 7,000
C, capital 3,000
Total 30,000

Profit and loss ratio is 3:2:1 for A, B and C, respectively. The other noncash assets were realized as follows:
Date Cash Received Book value
January 2013 6,000 9,000
February 2013 3,500 7,700
March 2013 12,500 11,300

Cash is distributed as other noncash assets realized.

Total cash received by B is:


a. 0
b. 1,500
c. 2,000
d. 5,000
____ 6. Following is the income statement of XYZ Branch in Cebu
City Company, for the six months period ending June 30, 2020:

Sales 620,000
Cost of sales:
Shipments from home office 550,000
Purchases 50,000
Total 600,000
Inventory, June 30, 2020:
From Home Office 75,000
From purchases 10,000 85,000 515,000
Gross margin 105,000
Expenses 85,000
Net income for the month 20,000

The Home Office ships merchandise to, and bills the Branch Office at 125% of cost.

The rent of the Branch office fro six months at a monthly rate of 1,000 was paid by the home.

The Home office net profit from its Branch Office in Cebu City for the six (6) months ending June 30, 2020 is:
a. 14,000
b. 109,000
c. 125,000

Page 9 of 76
d. 139,000
____ 7. Mexican Builders, Inc. employs the cost-to-cost method in determining the percentage of
completion for revenue recognition. The company’s records show the following information on a recently
completed project for a contract price of P5,000,000.

2018 2019 2020


Costs incurred to date 900,000 2,550,000 ?
Gross profit (loss) 100,000 350,000 (50,000)

The estimated costs to complete the project at December 31, 2019:


a. 850,000
b. 1,700,000
c. 2,300,000
d. 2,550,000
____ 8. Albaros Textile Company has a single branch in Bulacan.
On March 2, 2013, the home office accounting records included an Allowance for Overvaluation of
Inventories - Bulacan branch ledger account with a credit balance of 32,000. During March, merchandise
costing 36,000 was shipped to the Bulacan branch and billed at a price representing a 40% markup on the
billed price. On March 31, 2013, the Branch prepared an Income statement indicating a net loss of 11,500 for
March and ending inventories at billed prices of 25,000. What is the amount of adjustment for Allowance for
Overvaluation of Inventories to reflect the true branch net income?
a. 39,257 debit
b. 46,000 credit
c. 39,333 debit
d. 46,000 debit
____ 9. The balance sheet, as of June 39, 2013, for the partnership of
DD, JJ and RR shows the following information:

Assets 360,000 DD, loan 20,000


DD, capital 83,000
JJ, capital 77,000
RR, capital 180,000
Total 360,000 Total 360,000

It was agreed among the partners that DD retires from the partnership, and it was also further agreed that the
assets should be adjusted to their fair value of P408,000 as of June 30, 2013. The partnership is to pay DD
P121,000 cash for DD’s partnership interest, which would include be payment of his loan. No goodwill is to be
recorded. DD, JJ and RR share profit 25%, 25% and 50% respectively.

After DD’s retirement, how much would RR’s capital be?


a. 360,000
b. 200,000
c. 180,000
d. 120,000
____ 10. Which of the following procedures is acceptable when accounting for a deficit balance in a
partner’s capital account during partnership liquidation?
a. A partner with a negative capital balance must contribute personal assets to the partnership
that are sufficient to bring the capital account to zero.
b. If a partner with a negative capital balance is personally insolvent, the negative capital
balance may be absorbed by those partners having a positive capital balance according to
the residual profit and loss sharing ratios that apply to all the partners.
c. If a partner with a negative capital balance is personally insolvent, the negative capital
balance may be absorbed by those partners having a positive capital balance according to
the residual profit and loss sharing ratios that apply to those partners having positive
balances.
d. All of the procedures are acceptable.
____ 11. In partnership liquidation, how are partner salary allocations treated?
a. Salary allocations take precedence over creditor payments.
b. Salary allocations take precedence over amounts due to partners with respect to their
capital interests, but not profits.
c. Salary allocations take precedence over amounts due to partners with respect to their
capital profits, but not capital interests.

Page 10 of 76
d. Salary allocations are disregarded
____ 12. Under the rule of offset, what is the proper disposition of a partnership loan that was made
from a partner who has a debit balance?
a. The loan is first paid to the debtor partner before cash payments are made to partners.
b. The loan is written off as a partnership loss if the partner does not have the cash to cover
the debit balance.
c. The loan is charged off to the capital accounts of all the partners in their profit and loss
sharing ratios.
d. The loan is charged off to the capital account of the debtor partner.
____ 13. In a schedule of assumed loss absorptions
a. the partner with lowest loss absorption is eliminated last
b. it is necessary to have a cash distribution plan first
c. the least vulnerable partner is eliminated first
d. the most vulnerable partner is eliminated first
____ 14. Which partner is considered the most vulnerable as a result of a computation of vulnerability
rankings?
a. The partner with the lowest vulnerability ranking, who also has the lowest loss absorption
potential.
b. The partner with the lowest vulnerability ranking, who also has the highest loss absorption
potential.
c. The partner with the highest vulnerability ratio, who also has the lowest loss absorption
potential.
d. The partner with the highest vulnerability ranking, who also has the highest loss
absorption potential.
____ 15. If all partners are included in the first installment of an installment liquidation, then in future
installments
a. cash will be distributed according to the residual profit and loss sharing ratio.
b. cash should not be distributed until all non-cash assets are converted into cash.
c. a safe payments schedule must be prepared before each cash distribution to avoid
excessive payments to partners.
d. a cash distribution plan must be prepared so that partners will know when they will be
included in cash distributions

Jose, Maria, and Jenny are in the process of liquidating their partnership. Jenny has agreed to accept the
inventory, which has a fair value of P60,000, as part of her settlement. A balance sheet and the residual profit
and loss sharing percentages are as follows:

Cash 198,000 Accounts payable 149,000


Inventory 80,000 Jose, capital (40%) 79,000
Plant assets 230,000 Maria, capital (40%) 140,000
Jenny, capital (20%) 140,000

Total assets 508,000 Total liab./equity 508,000

____ 16. If the partners then distribute the available cash, Jose will receive
a. 23,000
b. 29,000
c. 0
d. 34,000
____ 17. USC, a nonprofit university, received the following cash contributions from donors during
the year 2014:

Unrestricted contributions 250,000


Contributions restricted by donors for scholarship programs 100,000
Contributions from a donor who stipulated that the money be spent in
accordance to the wishes of the hospital’s board of trustees 75,000
Contributions restricted by donors for equipment acquisitions 125,000

Assuming the university spent P 75,000 of the donors' contributions for scholarship programs on financing
this year' scholars, how much should be included in its current funds revenue for the year ended December 31,
2014?
a. 350,000

Page 11 of 76
b. 325,000
c. 400,000
d. 250,000
____ 18. Which of the following transactions will increase the normal balance of home office account
in the separate statement of financial position of the branch?
a. Collection by the home office of branch’s receivable
b. Debit memo received from the home office
c. Credit memo issued by the home office
d. Payment by the branch of home office’s loans payable
____ 19. Baste Company owns an 80% controlling interest in the Bastion Company. Bastion regularly
sells merchandise to Baste, which then sold to outside parties. The gross profit on all such sales is 40%. On
January 1, 2016, Baste sold land and a building to Bastion. The value of the parcel is 20% to land and 80% to
structures. The data are the following:

Baste Bastion
Internally generated net income, 2016 1,560,000 750,000
Internally generated net income, 2017 10,320,000 705,000
Intercompany merchandise sales, 2016 300,000
Intercompany merchandise sales, 2017 360,000
Intercompany inventory, December 31, 2016 45,000
Intercompany inventory, December 31, 2017 60,000
Cost of real estate sold on January 1, 2016 1,800,000
Sales price of real estate on January 1, 2016 2,400,000
Depreciable life of building 20 years

For 2016, what is the consolidated comprehensive income attributable to controlling interest?
a. 1,569,600
b. 1,575,000
c. 1,875,000
d. 1,597,500
____ 20. Jane had the following information

1. Purchased merchandises from a foreign supplier on January 20, 2016 for the Philippine peso equivalent of
P60,000 and paid the invoice on April 20, 2016 at the Philippine peso equivalent of P68,000.

2. On September 1, 2016, borrowed the Philippine peso equivalent of P300,000 evidence by a note that is
payable in the lender’s local currency on September 1, 2017. On December 31, 2016, the Philippine peso
equivalent of the principal amount was P320,000.

In Jane’s income statement, what amount should be included as a foreign exchange loss?
a. 4,000
b. 20,000
c. 22,000
d. 28,000
____ 21. Which of the following is correct?
a. Where the joint operators have designed the joint arrangement so that its activities
primarily aim to provide the parties with an output it will be classified as a joint control.
b. All joint arrangements are not structured through a separate vehicle are classified as joint
ventures.
c. For a joint venture, the rights pertain to the rights and obligations associated with
individual assets are liabilities, where with a joint operation, the rights and obligations
pertain to the net assets.
d. In considering the legal form of the separate vehicle if the legal form establishes right to
individual assets and obligations, the arrangement is a joint operation. If the legal form
established right to net assets of the arrangement, then the arrangement is a joint venture.
____ 22. Rommel, Inc acquired a 60% interest in Mikee Company several years ago. During 2014,
Mikee sold inventory costing P75,000 to Rommel for P100,000. A total of 16% of this inventory was not sold
to outsider until 2015. During 2015, Mikee sold inventory costing P96,000 to Rommel for P120,000. A total
of 35% of this inventory was not sold to outsiders until 2016. In 2015, Rommel reported cost of sales of
P380,000 while Mikee reported P210,000. What is the consolidated cost of sales?
a. 594,400
b. 473,440

Page 12 of 76
c. 474,400
d. 522,400
____ 23. Patrick Company acquired the assets (except for cash) and assumed the liabilities of Steve
Company on January 2, 2014 and Steve Company is dissolved. As compensation, Patrick Company gave
24,000 shares of its common stock, 12,000 shares of its 8% preferred stock , and cash of P240,000 to the
stockholders of Steve Company. On the date of acquisition, Patrick Company had the following
characteristics:

Common, par value P5; fair value, P20


Preferred, par value P100; fair value, P100

Immediately prior to acquisition, Steve Company’s balance sheet was as follows:

Cash 132,000
Accounts receivable (net of P4,000 allowance) 170,000
Inventory 200,000
Land 384,000
Building and equipment, net 1,032,000
Total 1,918,000

Current liabilities 228,000


Bonds payable, 10% 400,000
Common stock, P5 par value 600,000
Additional paid in capital 380,000
Retained earnings 310,000
Total 1,918,000

An appraisal of Steve Company showed that the fair values of its assets and liabilities were equal to their
book values except for the following, which had fair values as indicated:

Accounts receivable 158,000


Inventory 412,000
Land 540,000
Bonds payable 448,000

How much must be the goodwill recognized as a result of this business combination?
a. 322,000
b. 454,000
c. 94,000
d. 0
____ 24. The Jake Department is the first of a two-stage production process. Spoilage is identified
when the units have completed the Jake process. Cost of spoiled units are assigned to units completed and
transferred to the second department in the period spoilage is identified. The following information concerns
Jake’s conversion cots in May 2014:

Unit Conversion costs


Beginning work in process (50% complete) 2,000 P10,000
Units started during May 8,000 75,500
Spoilage - normal 500
Units completed and transferred 7,000
Ending work in process (80% complete) 2,500

Using the average method, what was Jake’s conversion cost transferred to the second department?
a. 59,850
b. 64,125
c. 67,500
d. 71,250
____ 25. Omni Company uses a job order cost system and has two production departments, T and P.
Budgeted information for the year is as follows:

Department T Department P
Machine hours 500 25,000

Page 13 of 76
Direct materials P400,000 P600,000
Direct labor 350,000 100,000
Factory overhead 455,000 300,000

Both Department T and Department P apply factory overhead to production orders through the use of
predetermined factory overhead application rates, which are based upon the yearly budget. Department T
applies factory overhead on a direct labor cost basis while Department P does so on a machine hours basis.
Actual information relating to Job 194 during the year was as follows:

Department T Department P Total


Machine hours 150 2,500 2,650
Direct materials P18,000
Direct labor P11,000 P4,500 P15,500
Factory overhead control P14,500 P24,600 P39,100

If Omni Company contracted to sell Job 194 for P100,000, and if estimated selling and administrative
expenses are 5% of the selling price, what is the estimated profit on Job 194?
a. 17,200
b. 22,400
c. 28,600
d. 33,700
____ 26. On July 1, 2015, DMCI Construction Company contracted to build an office building for RH
Corporation for a total contract price of P9.75 million. On July 1, Mean estimated that it would take between
2 to 3 years to complete the building. On December 31, 2017, the building was deemed substantially
completed. Following are accumulated contract costs incurred, total estimated costs, and accumulated billings
to RH for 2015, 2016, and 2017.

12/31/2015 12/31/2016 12/31/2017


Contract costs incurred to date 750,000 6,000,000 10,500,000
Total estimated costs 7,500,000 10,000,000 -
Billings to RH 1,500,000 5,500,000 9,250,000

Using the percentage of completion method, determine the correct income (loss) from construction to be
presented in the income statement of the company for the years 2015, 2016 and 2017, respectively.
a. P225,000; (P250,000); (P500,000)
b. P225,000; (P225,000); (P750,000)
c. P225,000; (P475,000); (P500,000)
d. P225,000; P250,000; (P750,000)
____ 27. Which of the following is false?
a. The FIFO method of process costing when beginning and ending work in process
inventories are each 50% complete.
b. For a manufacturing company, indirect manufacturing costs may be included in both work
in process inventory and finished goods inventory
c. A discrete loss is assumed to occur at specific point in the production process
d. Abnormal spoilage is always accounted for an equivalent unit basis.
____ 28. This interpretation provides the accounting principles for recognizing and measuring the
obligations and related rights in service concession arrangements. The issues addressed are

I Operating services
II Borrowing costs
III Subsequent accounting treatment of a financial asset and an intangible asset, and
items provided to the operator by the grantor.

a. I and II only
b. II and III only
c. I and III
d. I, II and III
____ 29. DMCI has used the cost-to-cost percentage of completion method of recognizing profits.
Tony assumed leadership of the business after the recent death of his father Ton. In reviewing the records,
Tony finds the following information regarding a recently completed building project for which the total
contract price was P50 million.

Page 14 of 76
Construction in progress account balance 2015 10,000,000
Construction cost incurred during 2017 20,500,000
Gross profit (loss) recognized in 2015 1,000,000
Gross profit (loss) recognized in 2016 3,500,000
Gross profit (loss) recognized in 2017 (500,000)

How much cost was incurred in 2016?


a. 16,500,000
b. 25,500,000
c. 9,000,000
d. 46,000,000
____ 30. Edwin Corporation is a manufacturing company engaged in the production of a single special
product known as “Toy”. Production costs are accumulated with the use of a job-order cost system.

The following information is available as of June 1, 2020:

Work in process 10,710


Direct materials inventory 48,600

In analyzing the job order cost sheets, the records disclosed that the compositions of the work in process
inventory on June 1, 2020 were as follows:

Direct materials used 3,960


Direct labor (900 hours) 4,500
Factory overhead applied 2,250

The following manufacturing activity occurred during the month of June 2020:

Purchased direct materials costing P60,000


Direct labor worked 9,900 hours at P5 per hour
Factory overhead of P2.50 per direct labor hour was applied to production.

At the end of June 2020, the following information was gathered in connection with the inventories

Inventories of work in process


Direct materials used 12,960
Direct labor (1,500 hours) 7,500
Factory overhead applied 3,750
24,210
Inventory of direct materials 51,000

Compute the cost of goods manufactured:


a. 142,560
b. 118,350
c. 131,850
d. 108,600
____ 31. The income statement submitted by the Balanga Branch to the Home Office for the month of
December, 2020 is shown below. After effecting the necessary adjustments the true net income of the branch
was ascertained to be P156,000.

Sales 600,000
Cost of sales:
Inventory, December 1 80,000
Shipments from Home Office 350,000
Local purchases 30,000
Total available for sale 460,000
Inventory, December 31 100,000 360,000
Gross margin 240,000
Operating expenses 180,000
Net income 60,000

Page 15 of 76
The branch inventories were:

12/1/2020 12/31/2020
Merchandise from home office 70,000 84,000
Local purchases 10,000 16,000
Total 80,000 100,000

The billing price based on cost imposed by the home office to the branch and
a. 140%
b. 100%
c. 40%
d. 29%
____ 32. In 2018, Megaworld, began construction work under a 3-year contract. The contract price
was P800,000. Megaworld uses the percentage of completion method for financial accounting purposes. The
income to be recognized each year is based on the proportion of costs incurred to total estimated costs for
completing the contract. The financial statement presentation relating to this contract at December 31, 2018
were as follows:

Balance sheet
Accounts receivable - construction contract billings 17,200
Construction in progress 52,000
Less: Contract billings 49,200
Cost of uncompleted contract in excess of billings 2,800
Income statement
Gross profit (before tax) recognized in 2018 14,560

What was the initial estimated gross before tax on this contract?
a. 52,000
b. 224,000
c. 576,000
d. 800,000
____ 33. Jun Industries acquired an 80% interest in Jamie Company by purchasing 24,000 of its
30,000 outstanding shares of common stock at book value of P105,000 on January 1, 2014. Jamie reported net
income in 2014 of P45,000 and in 2015 of P60,000 earned evenly throughout the respective years. Jun
received P12,000 dividends from Jamie in 2014 and P18,000 in 2015. Jun uses the equity method to record its
investment.

The balance of Jun’s investment in Jamie account at December 31, 2015 is:
a. 105,000
b. 138,600
c. 159,000
d. 165,000
____ 34. Oscar Corporation had a realized foreign exchange loss of P15,000 for the year ended
December 31, 2019 and must also determine whether the following items will require year-end adjustment:

1. Oscar had an P8,000 loss resulting from the translation of the accounts of its wholly owned foreign
subsidiary for the year ended December 31, 2019.

2. Oscar had an account payable to an unrelated foreign supplier payable in the supplier’s local currency. The
Philippine peso equivalent of the payable was P64,000 on the October 31, 2019 invoice date, and it was
P60,000 on December 31, 2019. The invoice is payable on January 20, 2020.

In Oscar’s 2019 consolidated income statement, what amount should be included as foreign exchange loss?
a. 11,000
b. 15,000
c. 19,000
d. 23,000
____ 35. On December 31, 2015, the unadjusted balance of investment in Butuan Branch Account is
P165,920 in the Home Office’s Book. The following data were found in your examination of the books of the
Home Office and its Butuan Branch:

1. Transfer of fixed assets from Home Office amounting to P53,960 was not booked by Butuan Branch.

Page 16 of 76
2. P10,000 covering marketing expense of Bataan Branch was charged by Home Office to Butuan Branch.

3. Butuan Branch recorded a debit note on inventory transfers from Home Office of P75,000 twice.

4. Home Office recorded cash transfer of P65,700 from Butuan Branch as coming from Davao Branch.

5. Butuan Branch reversed a previous debit memo from Bohol Branch amounting to P10,500. Home Office
decided that this charge is appropriately Davao’s Branch.

6. Butuan Branch recorded a debit memo from Home Office of P4,650 as P4,560.

What is the unadjusted balance of Home Office Current Account in Butuan Branch’s Books on December 31,
2015?
a. 92,336
b. 98,230
c. 104,500
d. 111,170
____ 36. On January 1, 2020, Jeszie’s Grill Inc. granted a franchise to Ms. Jessie Sarmiento to sell
Jeszie’s products. The franchise agreement provides the following terms:

As of December 31, 2020, Jeszie’s Grill Inc has not yet performed substantially all material services or
conditions required of the franchise contract. For the year ended December 31, 2020, Ms. Jessie Sarmiento
reported sales revenue in the amount of P1,000,000.

What is the amount of total income to be reported by Jeszie’s Grill for the year ended December 31, 2020?
a. 0
b. 50,000
c. 12,631,574
d. 808,157
____ 37. Post, Inc. had a receivable from a foreign customer that is payable in the customer’s local
currency. On December 31, 2010, Post correctly included this receivable for 200,000 local currency units in
its balance sheet at P110,000. When Post collected the receivable on February 15, 2011, the Philippine peso
equivalent was P95,000. In Post’s 2011 consolidated income statement, how much should it report as a
foreign exchange loss?
a. 0
b. 10,000
c. 15,000
d. 25,000
____ 38. Karen and Andrea are currently changing their partnership profit and loss ratios from 75/25 to
60/40. They have created a list of assets that have market and book value differences. One of the assets is a
building with a P300,000 market value and P200,000 book value. Two years after changing the profit and loss
ratios, the building is sold for P380,000. How much of the profit is allocated to Karen?
a. 108,000
b. 123,000
c. 135,000
d. 183,000
____ 39. A chemical company manufactures joint products Pep and Vim, and a by-product Zest. Costs
are assigned to the joint products by the market value method, which considers further processing costs in
subsequent operations. For allocating joint costs to the by-product, the market value or reversal cost method is
used. The total manufacturing costs for 10,000 units were P172,000 during the quarter. Production and cost
date follow:

Pep Vim Zest


Units produced 5,000 4,000 1,000
Sales price per unit P50 P40 P5
Further processing cost per unit 10 5 -
Selling and administrative expense per unit 2
Operating profit per unit 1

Compute the gross profit for Pep:


a. 0
b. 70,000

Page 17 of 76
c. 80,000
d. 100,000
____ 40. A partnership has the following balance sheet just before the final liquidation is to begin:

Cash 26,000 Liabilities 50,000


Inventory 31,000 Art Capital (40%) 18,000
Other assets 62,000 Raymond Capital (30%) 25,000
Darby Capital (30%) 26,000
Total 119,000 Total 119,000

Liquidation expenses are estimated to be P12,000. The other assets are sold for P40,000. What distribution
can be made to the partners?
a. Art (1,333); Raymond (1,333); Darby (1,334)
b. Art (0); Raymond (1,500); Darby (2,500)
c. Art (600); Raymond (1,200); Darby (2,200)
d. Art (0); Raymond (1,200); Darby (2,800)

Page 18 of 76
Mock Exam: AFAR
Answer Section

MULTIPLE CHOICE

1. B
SOL:
Market value of repossessed merchandise 3,000
Less: Unrecovered cost
Installement accounts receivable - 2019, unpaid balance 7,750.00
Less: Deferred gross profit -2019 (54/120 x P7,750) 3,487.50 4,262.50
Loss on repossession (1,262.50)
2. B
SOL:
One of the problems that may arise in measuring the assets and liabilities of the acquiree is that the initial
accounting for the business combination may be incomplete by the end of the reporting period.

For example, the acquisition date may be August 18 and the end of the reporting period may be August 31. In
this situation, in accordance with par. 45, the acquirer must report provisional amounts in its financial
statements. The provisional amounts will be best estimates and will need to be adjusted to fair values when
those amounts can be determined after the end of the reporting period.The measurement period in which the
adjustments can be made cannot exceed one year after the acquisition date.
3. B
SOL:
Acquisition price
Under the acquisition method, contingent consideration obligators are recognized as part of the initial value
assigned in a business combination, consistent with fair value concept. Therefore, the acquiring firm must
estimate the fair value of the contingent portion of the total business fair value. The contingency’s fair value is
recognized as part of the acquisition regardless of whether it is based on future performance of the target firm
or the future stock prices of the acquirer.

Fair value of consideration transferred:


Cash paid 400,000
Add: Contingent consideration, present value 3,142
Consideration transferred/acquisition price 403,142

The loss from contingent performance obligation is reported in Morris Inc.’s consolidated income statement
as a component of ordinary income.
4. D
SOL:
A B C D Total
Capital balances 16,200 12,000 37,700 17,700 83,600
Loans 12,000 14,400 - 9,600 36,000
Salaries - - 160 240 400
Total interests 28,200 26,400 37,860 27,540 120,000
Reduction in interests (equally) (27,750) (27,750) (27,750) (27,750) (111,000)
Balances 450 (1,350) 10,110 (210) 9,000 (a)
Absorption of possible insolvency (780) 1,350 (780) 210 -
Balances (330) - 9,330 - 9,000
Absorption of possible insolvency 330 - (330) - -
Payment - - 9,000 - 9,000

(a) Payment to partners:


Cash, beginning 6,000
Proceeds (18,000 - 2,400) 15,600
Payment of liabilities (always in full) (12,400)
Payment of liquidation expenses (600)
9,000
5. D
SOL:
A B C Total
Capital balances 12,500 7,000 3,000 12,500
Loans 2,500 - - 2,500

Page 19 of 76
Total Interests 15,000 7,000 3,000 25,000
Reduction in interests (3:2:1) (11,000) (7,333) (3,667) (22,000)
Balances 4,000 (333) (667) 3,000
Reduction in interests (1,000) 333 667 -
Payments (a) 3,000 - - 3,000

(a) January:
Cash, beginning 2,000
Add: Proceeds 6,000
Less: Payments of liabilities 5,000
Payment to Partners 3,000 (a)

A B C D
Total Interests - January 15,000 7,000 3,000 25,000
Payment to partners - January (3,000) - - (3,000)
Total interest - February 12,000 7,000 3,000 22,000
Reduction in interests (3:2:1) (9,250) (6,167) (3,083) (18,500)
Balances (b) 2,750 833 (83) 3,500
Reduction in interests (50) (33) 83 -
Payments 2,700 800 - 3,500

(b) February:
Cash, beginning 0
Add: Proceeds 3,500
Less: Payments of liabilities 0
Payments to partners 3,500 (b)

A B C D
Total interest - February 12,000 7,000 3,000 22,000
Payment to partners - February (2,700) (800) - (3,500)
Total interests - March 9,300 6,200 3,000 18,500
Reduction in interests (3:2:1) (3,000) (2,000) (1,000) (6,000)
Balances (c) 6,300 4,200 2,000 12,500

(c) March:
Cash, beginning 0
Add: Proceeds 12,500
Less: Payments of liabilities 0
Payment to Partners 12,500 (c)

Total payment to Partner B:


B, Capital 7,000
Less: Share in total realization loss: 2/6 x 6,000 2,000
Total cash received by B 5,000

Therefore, total payment to B should be:


January 0
February 800
March 4,200
5,000
6. B
SOL:
Net Income as reported by the Branch 20,000
Less: Rental expense charged by the home office (1,000 x 6 months) 6,000
Adjusted NI as reported by the branch 14,000
Add: Overvaluation of Cost of goods sold
MI beginning -
SFHO 550,000
Cost of goods available for sale 550,000
Less: Inventory, ending 75,000
Cost of goods sold, at BP 475,000
x Mark-up ratio 25/125 95,000

Page 20 of 76
Real Branch Net Income 109,000
7. A
SOL:
2018 2019 2020
Contract price 5,000,000 5,000,000 5,000,000
Cost incurred each year - - 2,050,000
Add: Cost incurred in prior year - 900,000 2,550,000
Costs incurred to date 900,000 2,550,000 4,600,000
Add: estimated costs to complete - 1,700,000 -
Total estimated costs - 4,250,000 4,000,000
Estimated gross profit - 750,000 400,000
Multiply by: percentage of - 60% 100%
completion
Recognized gross profit to date 100,000 450,000 400,000
Less: Recognized gross profit in prior 100,000
years - 450,000
Recognized gross profit each year 100,000 350,000 (50,000)
8. D
SOL:
100% 60% 40%
Billed Price Cost Allowance
Merchandise inventory, 1/1/05 32,000
Shipments *60,000 36,000 *24,000
Cost of goods available for sale 56,000
Less: MI, 3/31/05 (25,000 x 40%) 10,000
Overvaluation of CCS/RBP sales 46,000

*36,000 cost /60% = 60,000 x 40% = 24,000, (Note : mark up is based on billed price)
9. B
SOL:
DD: P83,000 + P20,000 + (P408,000 - P360,000) x 25% = P115,000
JJ: P77,000 + (P408,000 - P360,000) x 25% = P89,000 - (P6,000 x 25/75) = P87,000
RR: P180,000 + (P408,000 - P360,000) x 50% = P204,000 - (P6,000 x 50/75) = P200,000

Amount paid to DD 121,000


Less: Book value of DD’s interest 115,000
Bonus to retiring partner 6,000
10. C
11. D
12. B
13. D
14. A
15. A
16. C
SOL:
40% 40% 20%
Jose Maria Jenny
Equities 79,000 140,000 140,000
Distribute inventory to Jenny and: (60,000)
recognize P20,000 loss (8,000) (8,000) (4,000)
Possible losses on plant (92,000) (92,000) (46,000)
Subtotal (21,000) 40,000 30,000
Eliminate Jose’s debit
balance to Maria & Jenny 21,000 (14,000) (7,000)
Balance 0 26,000 23,000
17. C
SOL:
Unrestricted contributions 250,000
Contributions from a donor who stipulated that the money be spent in
accordance to he wishes to the hospital’s board of trustees 75,000
Contributions used for scholarship 75,000
Current fund revenue 400,000

Page 21 of 76
18. B
19. A
SOL:
Internally generated net income, 2016 - parent 1,560,000
Gain on sale of real estate, 1/1/2016 (60,000)
Realized gain, 12/31/2016 ((80% x P600,000)/20) 24,000
Adjusted internally generated net income 984,000
Internally generated net income, 2016
Subsidiary 750,000
Unrealized profit in ending inventory (40% x P45,000) (18,000) 732,000
Consolidated net income 1,716,000
NCI net income (20% x P732,000) 146,400
Attributable to controlling interest 1,569,600
20. D
SOL:
Loss (60,000 - 68,000) 8,000
Loss (300,000 - 320,000) 20,000
Total 28,000
21. D
22. C
SOL:
Cost of sales of Rommel 380,000
Cost of sales of Mikee 210,000
Less: Intercompany sales 2015 (120,000)
Mark-up on beginning inventory (100,000 - 75,000) x 16% (4,000)
Add: Mark-up on ending inventory (120,000 - 96,000) x 35% 8,400
Consolidated cost of sales 474,400
23. B
SOL:
Cost of investment
Common shares (24,000 x 20) 480,000
Preferred shares (12,000 x 100) 1,200,000
Cash 240,000 1,920,000
Fair value of identifiable net assets acquired
Accounts receivable 158,000
Inventory 412,000
Land 540,000
Buildings and equipment 1,032,000
Current liabilities (228,000)
Bonds payable (448,000) 1,466,000
Goodwill from business combination 454,000
24. C
SOL:
Equivalent production
Units completed and transferred 7,000
Ending work in process (2,500 x 80%) 2,000
Spoilage - normal 500 9,500
Costs 85,500
Unit costs 9.00

Cost accounting
Completed and transferred
Original cost (7,000 x P9,00) 63,000
Add: Cost of normal spoilage (500 x P9) 4,500
67,500
25. A
SOL:
Selling price 100,000
Cost of goods sold:
Direct materials cost 18,000
Direct labor cost (11,000 + 4,500) 15,500
Factory overhead

Page 22 of 76
Dept. T (455,000/350,000 x 11,000) 14,300
Dept. P (300,000/25,000 x 2,500) 30,000 77,800
Selling and administrative expenses (5% x 100,00) 5,000
Net profit 17,200
26. C
SOL:
2015 2016 2017
Contract price 9,750,000 9,750,000 9,750,000
Total estimated costs 7,500,000 10,000,000 10,500,000
Estimated (actual) profit (loss) 2,250,000 (250,000) (750,000)
Percentage of completion
750,000/7,500,000 10%
Recognized in full 100% 100%
Gross profit to date 225,000 (250,000) (750,000)
Less: Gross profit (loss) prior year - 225,000 (250,000)
Gross profit (loss) during the year 225,000 (475,000) (500,000)
27. A
28. D
29. A
SOL:
Contract price 5,000,000
Gross profit recognized:
2015 1,000,000
2016 3,000,000
2017 (500,000) 4,000,000
Total cost incurred 46,000,000
Less: Incurred in 2015
Construction in process 10,000,000
Less: Gross profit recognized (1,000,000) (9,000,000)
Incurred in 2017 (20,500,000)
Cost incurred in 2016 16,500,000
30. B
SOL:
Direct materials inventory, June 1, 2020 48,600
Add: Purchases 60,000
Direct materials available for use 108,600
Less: Direct materials inventory, June 30, 2020 51,000
Direct materials used 57,600
Direct labor (9,900 hours x P5 per hour) 49,500
Applied factory overhead (9,900 hours x P2.5 per hour 24,750
Manufacturing cost 131,850
Add: Work in process, June 1, 2020 10,710
Total work placed in process 142,560
Less: Work in process, June 30, 2020 24,210
Cost of goods manufactured 118,350
31. A
SOL:
Billing price based on cost
True branch net income 156,000
Less: Branch net income as reported (by the branch) 60,000
Overvaluation of cost of goods sold 96,000
Less: Cost of goods sold from home office at billed price
Inventory, December 1 70,000
Shipments from home office 350,000
Cost of goods from home office available for sale 420,000
Less: Inventory, December 31 84,000 336,000
Cost of goods sold from home office, at cost 240,000

Billed price (P336,000/P240,000) 140%

Allowance for overvaluation after adjustment:


P84,000 x 40/140 = P24,000

Page 23 of 76
32. B
SOL:
Contract price 800,000
Cost incurred each year 37,440
Add: Cost incurred in prior years -
Costs incurred to date 37,440
Add: Estimated cost to complete
Total estimated costs
Estimated gross profit (loss) 224,000
Multiply by: % of completion (14,560 + 37,440) / 6.5%
800,000
Recognized gross profit (loss) to date 15,000
Less: Gross profit (loss) in prior year -
Recognized gross profit (loss) in current year 14,560
33. C
SOL:
Investment, 1/1/2014 105,000
Add: Share in net income, 2014 (P45,000 x 80%) 36,000
Less: Dividends received 12,000
Investment, 12/31/2014 129,000
Add: Share in net income, 2015 (P60,000 x 80%) 48,000
Less: Dividends received 18,000
Investment, 12/31/2015 159,000
34. A
SOL:
Foreign exchange loss before adjustments 15,000
Gain on accounts payable, buyer (64,000 - 60,000) (4,000)
Adjusted foreign exchange loss in the income statement 11,000

The P8,000 loss resulting from translation of a subsidiary is presented at the stockholders’ equity section of
the consolidated balance sheet.
35. D
SOL:
Transaction Home Office Investment in Branch
Unadjusted balances 111,170 165,920
1 (53,960)
2 (10,000)
3 75,000
4 (65,700)
5
6 (90)
Adjusted balances 90,220 90,220
36. D
SOL:
Initial franchise fee = No revenue because of non-compliance with essential elements from revenue
recognition.

Contingent franchise fee (5% x 1,000,000) 50,000


Interest income for 2020 (7,581,573 x 10%) 758,157
Total income for the year ended December 31, 2020 808,157
37. C
SOL:
The peso value of the local currency unit receivable has decreased from P110,000 at December 31, 2010 to
P95,000 at February 15, 2011. This decrease of P15,000 should be reported as a foreign exchange loss in
2011.
38. B
SOL:
(300,000 - 200,000) (.75) + (380,000 - 300,000) (.60) = 123,000
39. D
SOL:
Pep: 5,000 x (P50-P10) = 200,000 x 50% =100,000
Vim: 4,000 x (P40-P5)=140,000

Page 24 of 76
Joint costs 172,000
Less: Joint costs allocated to by-product 2,000
Joint costs to joint products 170,000
Sales of Pep (P50 x 5,000) 250,000
Less: Cost of sales
Joint costs 100,000
Further processing cost 50,000
Gross profit 100,000
40. ANS: B
SOL:
Art Raymond Darby Total
Reported balances 18,000 25,000 26,000 69,000
Possible loss (26,000) (19,500) (19,500) (65,000)
(8,000) 5,500 6,500 4,000
Possible insolvency (3:3) 8,000 (4,000) (4,000) 0
Payment to partners - 1,500 2,500 4,000

Page 25 of 76
AUDITING
FIFTH YEAR.

____ 1. Which of the following statements regarding analytical procedures is not correct?
a. For certain accounts with small balances, analytical procedures alone may be sufficient
evidence.
b. Analytical procedures can be used as substantive tests.
c. Analytical tests emphasize a comparison of client internal controls to PFRS.
d. Analytical procedures are required on all audits.
____ 2. An auditor is obligated to communicate a proposed audit adjustment to an entity’s audit
committee only if the adjustment
a. results from the correction of a prior period’s departure from financial reporting standards
b. has a significant effect on the entity’s financial reporting process
c. is a recurring matter that was proposed to management in the prior year
d. has not been recorded before the end of the auditor’s field work
____ 3. Materiality at the account balance level is stated in planning an audit because
a. some users make decisions based upon individual account balances
b. the auditor verifies account balances in reaching an overall conclusion on the fairness of
the financial statements
c. the opinion on the fairness of the financial statements extends to the individual account
balances
d. official pronouncements have specified different levels of materiality for various financial
statement items.
____ 4. Management’s attitude towards aggressive financial reporting and its emphasis on meeting
projected profit goals most likely would significantly influence on entity’s control environment when
a. the audit committee is active in overseeing the entity’s financial reporting policies
b. internal auditors have direct access to the board of directors and entity management
c. management is dominated by one individual who is a also a shareholder
d. external policies established by parties outside the entity affects its accounting practices
____ 5. Which of the following is an objective of a review engagement?
a. reporting that the financial statements, in all material respects, fairly present the financial
position and operating results of the client.
b. reporting whether material modifications should be made to such financial statements to
make them conform with generally accepted accounting principles
c. expressing a positive opinion that the financial information is presented in conformity with
the generally accepted accounting principles
d. expressing a limited assurance to users who have agreed as to procedures that will be
performed by the CPA
____ 6. As the acceptable level of detection risk increases, an auditor may change the
a. assessed level of control risk from below the maximum to the maximum level
b. nature of substantive tests from a less effective to a more effective procedure
c. timing of substantive tests from year end to an interim date
d. assurance provided by tests of controls by using a larger sample size than planned
____ 7. When discussing control risk and the audit risk model, which one of the following statements
is false?
a. The relationship between control risk and detection risk is inverse.
b. If the auditor concludes that an internal control system is completely ineffective to prevent
or detect errors, he/she would assign 0% to control risk.
c. The relationship between control risk and evidence is direct.
d. Control risk is a measure of the auditor’s assessment of the likelihood that errors will not
be prevented or detected by the client’s internal control system.
____ 8. Which of the following statements is correct regarding an accountant’s working papers?
a. The clients owns the working papers but, in the absence of the accountant’s consent, may
not disclose them without a court order.
b. The accountants owns the working papers but generally may not disclose them without the
client’s consent or a court order.
c. The accountants owns the working papers and generally may disclose them as the
accountant sees fit.
d. The client owns the working papers but the accountant has custody of them until the
accountant’s bill is paid in full.

Page 26 of 76
____ 9. In designing audit programs, an auditor should establish specific audit objectives that related
primarily to the
a. financial statement assertions
b. timing of audit procedures
c. cost-benefit of gathering evidence
d. selected audit techniques
____ 10. If a lawyer refuses to furnish corroborating information regarding litigation, claims, and
assessments, the auditor should
a. seek to obtain the corroborating information from management
b. disclose this fact in the notes to financial statement
c. honor the confidentiality of the client-lawyer relationship
d. consider the refusal to be tantamount to a scope limitation
____ 11. Regarding litigation, the auditor’s primary means of obtaining corroboration of
management’s information is a _____.
a. confirmation of claims and assessments from an officer of the court presiding over the
litigation
b. confirmation of claims and assessments from the other parties to the litigation
c. letter of corroboration from the auditor’s lawyer upon review of the legal documentation
d. letter of audit inquiry to the client’s lawyer
____ 12. In line with your audit of Marlon Corporation for the period ended December 31, 2016, your
audit staff provided you the following audit notes:

1. Accounts receivables from customers increased during the year by P4,200,000. Total discounts taken by
customers was at P1,580,000 while total sales returns which included the customer refunds was at
P2,420,000.

2. The allowance for bad debts increase during the year by P840,000. During the year, the Company wrote-off
P1,120,000 in bad debts. While recovery of previous write-off (included in the cash collections from
customers) was at P420,000.

3. Advances from customers decreased during the year by P1,900,000.

4. Accounts payable to suppliers increased during the year by P3,780,000. Total discounts taken by the
Company for purchases was at P1,290,000 while total purchase returns which included the supplier refunds
was at P1,960,000.

5. Advances to suppliers increased during the year by P1,512,000.

6. Inventories increased during the year by P2,690,000.

7. The equipment account increased by P2,000,000 during the year while carrying value of the equipment sold
during the year was at P1,600,000.

8. The accumulated depreciation account increased by P1,000,000 during the year.

9. The following information had been provided by the company’s accountant based on its cash records:

Cash collections from customers 45,780,000


Cash payments to suppliers 24,490,000
Cash payment of operating expenses 8,650,000
Cash payment for acquisition of an equipment 5,000,000
Cash collection from disposal of an equipment 1,040,000
Cash refunds received from purchase returns 640,000
Cash refunds paid for sales returns 830,000

What is the accrual basis cost of sales?


a. 24,068,000
b. 25,580,000
c. 24,940,000
d. 23,428,000
____ 13. Which of the following substantive procedures should auditors ordinarily perform regarding
subsequent events?
a. Send second requests to the client’s customers who failed to respond to initial accounts
receivable confirmation requests.

Page 27 of 76
b. Compare the latest available interim financial statements with the financial statements
being audited.
c. Review the cut-off bank statements for several months after the year-
d. Communicate material weaknesses in internal control to the client’s audit committee
____ 14. Which of the following is not generally considered a financial statement audit risk factor?
a. rate of change in the entity’s industry is rapid
b. profitability of the entity relative to its industry is inconsistent.
c. a new client with no prior audit history
d. management operating and financing decisions are dominated by the top management
____ 15. Daryl Inc. had the following portfolio of financial assets as of December 31, 2015. All the
financial asset were acquired in 2015:

Financial asset Acquisition Cost


PLDT Corporation stocks, 20,000 shares 590,000
Meralco stocks, 40,000 shares 1,100,000
See Company, 10%. P2 million bonds 1,973,000
Miascor Corporation stocks, 50,000 shares 2,400,000

1. PLDT Corporation shares were acquired with an intention of generating short-term profits from the share
price’s fluctuations. The Company paid P29.50 per share, which included the P0.50 per share broker’s fees
and commissions. The shares were acquired on February 20, 2015. A P2 per share cash dividends were
received on March 30. These dividends were declared by PLDT Corporation on January 20, 2015 to
stockholders as of record date March 1, 2015.

2. The Company paid P27.50 per share, including P0.50 per share broker’s fees on the acquisition of Meralco,
Inc. on March 1, 2015. These shares were acquired for trading purposes. A P3 per share dividends were
received from the said shares on May 3, 2015. These dividends were declared on April 1 to stockholders as of
record date April 20.

3. See Company bonds which pay semi-annual interest every June 30 and December 31, were acquired on
October 1, 2015 at P1,973,000, when the prevailing effective interest rate on similar instrument was at 12%.
The bonds shall mature on December 31, 2017. The Company has a business model of holding debt securities
for short-term profits.

4. Miascor Corporation stocks were acquired P48 per share, including P3 per share broker’s fees and
commissions on June 30, 2015. Miascor Corporation had a total of 200,000 shares outstanding on the same
date. The company received P5 dividends per share from Miascor on December 20, 2015.

5. The following information were deemed relevant at year-end and no entries had Meralco made yet by the
company to reflect any of the following information:

PLDT Meralco Inc See Co. Miascor Corp.


Corp.
Net income in 2015 1,200,000 P1,500,000 P2,000,000 P2,240,000
Fair value P35/share P25/share 11% P51/share

What is the carrying value of investment in Miascor Corporation shares that should be presented in the 2015
Statement of Financial Position?
a. 2,550,000
b. 2,150,000
c. 2,430,000
d. 2,280,000
____ 16. Which of the following statements is correct concerning both an engagement to compile and
an engagement to review a non-public entity’s financial statements?
a. The CPA must be independent in fact and appearance
b. The CPA should obtain a written management representation letter.
c. The CPA expresses no assurance on the financial statements
d. The CPA does not contemplate obtaining an understanding of the internal control system
____ 17. When setting a preliminary judgment about materiality
a. the same amount of evidence is required for either low or high peso amounts
b. less evidence is required for a low peso amount than for a high peso amount
c. there is no relationship between it and the peso amount of evidence needed
d. more evidence is required for a low peso amount than for a high peso amount

Page 28 of 76
____ 18. An inaccurate form of the audit risk model would show that
a. detection risk is inversely related to inherent risk
b. increases in control risk will cause decreases in detection risk
c. detection risk can be determined from audit risk, inherent risk, and control risk
d. detection risk is inversely related to audit risk
____ 19. The expression “financial statements, taken as a whole” applies
a. equally to each material item in each financial statement
b. equally to a complete set of financial statements and to an individual financial statement
c. only to a complete set of financial statements
d. equally to each item in each financial statement
____ 20. Proper segregation of functional responsibilities in an effective structure of internal control
calls for separation of the functions of:
a. authorization, execution, and payment
b. custody, execution and reporting
c. authorization, recording and custody
d. authorization, payment and recording
____ 21. The Novak Company acquired several small companies at the end of 2013 and, based on the
acquisitions, reported the following intangibles in its December 31, 2013 statement of financial position:

Patent 200,000
Copyright 400,000
Tradename 350,000
Computer software 100,000
Goodwill 900,000

The Company’s accountant determines the patent has an expected life of 10 years and no expected residual
value, and that it will generate approximately equal benefits each year. The company expects to use the
copyright and tradename for the foreseeable future. The accountant knows that the computer software is used
in the company’s 120 sales offices. The company has replaced the software in 60 offices in 2014, and expects
to replace the software in 40 more offices in 2015 and the remainder in 2016.

On December 31, 2014, there are no indications of impairment of patent and computer software. The
following information relate to the other intangible assets:
1. Because of the rampant piracy, the copyright is expected to generate cash flows of just P8,000 per year.

2. The tradename is expected to generate cash flows of P15,000 per year.

3. The goodwill is associated with Novak Company’s Jade Production reporting unit. The cash flows
expected to be generated by the Jade Production reporting unit is P200,000 per year for the next 25 years. The
reporting unit has a carrying amount of P2,100,000 excluding goodwill.

Assume that the appropriate discount for all items is 5%.

What is the carrying amount of goodwill on December 31, 2014


a. 659,720
b. 900,000
c. 855,000
d. 718,789
____ 22. Preventing someone with sufficient technical skill from circumventing security procedures
and making changes to production programs is best accomplished by _____
a. reviewing of reports of job completed
b. running test data periodically
c. providing suitable segregation of duties
d. comparing production programs with independently controlled copies
____ 23. As part of obtaining an understanding of internal controls, an auditor is not required to
a. identify the types of potential misstatements that may occur
b. ascertain whether internal control policies and procedures have been placed operation.
c. consider factors tat affect the risk of material misstatement
d. obtain knowledge about the operating effectiveness of internal control
____ 24. Error in data processed in a batch computer system may not be detected immediately because
a. transaction trails in batch system are available only for a limited period of time.
b. errors in some transactions cause rejection of other transactions in the batch.

Page 29 of 76
c. there are time delays in processing transactions in a batch system.
d. random errors are more likely in a batch system than in an online system
____ 25. What is an example of specific transaction authorization?
a. establishment of a customer’s credit limits
b. setting of automatic reorder points
c. establishment of sales prices
d. approval of a construction budget for a new warehouse
____ 26. An audit of financial statements is conducted to determine if the
a. organization is operating efficiently and effectively
b. auditee is following specific procedures or rules set down by some higher authority
c. overall financial statements are stated in accordance with the reporting framework
d. client’s internal control is functioning as intended
____ 27. Regardless of how the allocation of the preliminary judgment about materiality was done,
when the audit is complete the auditor must be confident that the combined errors in all accounts are
a. less than or equal to the preliminary judgment
b. more than the preliminary judgment
c. less than the preliminary judgment
d. equal o the preliminary judgement
____ 28. Presented below is the statement of financial position of Roger Corporation prepared by the
accounting manager for the current year, 2020:

Roger Corporation
Statement of Financial Position
December 31, 2020
Current assets 435,000
Investments 640,000
PPE 1,720,000
Intangible assets 305,000
3,100,000
Current liabilities 330,000
Long-term liabilities 1,000,000
Shareholders’ equity 1,770,000
3,100,000

Consider the following information:


1. The current assets section includes: cash P100,000, accounts receivable P170,000 less P10,000 for
allowance for doubtful accounts, inventories P180,000, and unearned revenue P5,000. The cash balance is
composed of P114,000, less a bank overdraft of P14,000. Inventories are stated on the lower of FIFO cost or
market.

2. The investments section includes: the cash surrender value of a life insurance contract P40,000; investment
in ordinary shares, short-term (trading) P80,000 and long-term (available-for-sale) P270,000; bond sinking
fund P250,000. The cost and fair value of investments in ordinary shares are the same;

3. Property, plant and equipment includes: buildings P1,040,000 less accumulated depreciation P360,000;
equipment P450,000 less accumulated depreciation P180,000; land P500,000; and land held for future use
P270,000.

4. Intangible assets include: a franchise P165,000; goodwill P100,000; and discount on bonds payable
P40,000.

5. Current liabilities include: accounts payable P90,000, notes payable - short-term P80,000 and long-term
P120,000; and taxes payable P40,000.

6. Long-term liabilities are compose solely of 10% bonds payable due 2020.

7. Shareholders’ equity has: preference shares, no par value, authorized 200,000 shares, issued 70,000 shares
for 450,000; and ordinary shares, P1 par value, authorized 400,000 shares, issued 100,000 shares at an
average price of P10. In addition, the corporation has retained earnings of P320,000.

8. The company’s management does not elect to use the fair value option for any of its financial assets or
liabilities.

Page 30 of 76
What is the total assets?
a. 3,093,000
b. 2,979,000
c. 2,814,000
d. 3,079,000
____ 29. One example of a “Special Report” as defined by Philippine Standards on Auditing is a report
issued in connection with
a. price-level basis financial statements
b. compliance with a contractual agreement not related to the financial statements.
c. a feasibility study
d. a limited review of interim financial information
____ 30. ABC Company, a financing company, extended a loan to XYZ Corporation amounting to P10
million on January 1, 2011 receivable 5 years after. The loan bears 10% annual interest collectible at the end
of each year starting December 31, 2011. The Company paid direct origination cost amounting to P300,000
and charged XYZ Corporation origination fees at P1,020,955. The yield on the loan under this arrangement
was at 12%.

The 2011 to 2013 interests were collected as scheduled.

By the end of 2014, due to financial difficulties being experienced by XYZ, XYZ failed to pay the annual
interest as scheduled and ABC Company is doubtful as to the collectibility of the remaining interests and the
principal.

After due consideration and correspondence with XYZ Company, ABC estimated that it will be able to
recover the following amounts at respective estimated dates:

Amount Expected delivery date


1,000,000 December 31, 2015
2,000,000 December 31, 2016
2,500,000 December 31, 2017
2,500,000 December 31, 2018

What is the correct net book value of the receivables as of December 31, 2014?
a. 6,855,491
b. 5,855,491
c. 6,344,509
d. 5,344,509
____ 31. A purpose of a management representation letter is to replace
a. The scope of an auditor’s procedures concerning related party transactions and subsequent
events.
b. An auditor’s responsibility to detect material misstatements only to the extent that the
letter is relied on.
c. Audit risk to an aggregate level of misstatement that could be considered material.
d. The possibility of a misunderstanding concerning management’s responsibility for the
financial statements.
____ 32. Rafa Corporation, reported the following balances on the liability portion of its Statement of
Financial Position as of December 31, 2014:

Noncurrent liabilities
Note payable, bank 10% 4,500,000
Bonds payable, 12% 3,231,652
Finance lease liability 2,200,000

1. The note payable to the bank was originated on September 1, 2013 and is due annually at the rate of P1.5
million every August 31 starting 2014. Interest which is based on the outstanding balance of the loan is also
payable every August 31. Interest is yet to be accrued on the note by the balance sheet date. Payments on the
note and interest during the current year had Meralco recorded appropriately.

2. The 5 year, 12% bonds payable (with a face value of P3 million) were issued on January 1, 2014 at the
prevailing market rate of interest which is 10%. Interest on the bonds are payable semi-annually every June
30 and December 31. The entry made by the client to record the issuance was to debit cash and credit bonds
payable for the total consideration received.

Page 31 of 76
3. The lease liability is in relation to Rafa’s purchase of a machine on December 31, 2013. The machine was
delivered the same day to the company. The lease stipulates that annual payments will be made for 5 years
starting December 31, 2013. At the end of the 5-year term, the company may purchase the machine. The
estimated economic life of the machine is 12 years. Your further investigation revealed the following terms of
the transaction:

Annual lease payments 550,000


Purchase option price 250,000
Estimated fair value after 5 years 750,000
Implicit rate 10%
Borrowing rate 12%

Entry made:
Debit: Equipment (2,750,000) and credit: Cash (550,000) and Finance lease liability (2,200,00)

Payment on December 31, 2014 is yet to be recorded.

What is the correct total noncurrent liabilities to be presented in the 2014 Statement of Financial Position
a. 6,049,341
b. 5,842,069
c. 6,193,897
d. 7,336,271
____ 33. You were assigned to audit the stockholders’ equity accounts of Mico Corporation for the
period ended December 31, 2016. The Company’s equity balances as of December 31, 2015 are as follows:

Ordinary shares, P100 par value, 500,000 shares authorized, 80,000 8,000,000
shares issued
Preference shares, P50 par value, 200,000 shares authorized, 100,000 5,000,000
shares issued
Share premium - Ordinary shares 2,400,000
Share premium - Preference shares 1,000,000
Accumulated profits 3,890,000

Your audit examinations revealed the following transaction occurring in 2016:

1. On February 2, the Company reacquired for P1,168,000, 8,000 share of ordinary as treasury shares.

2. On April 5, 20,000 ordinary shares and 10,000 preference shares were issued for a lump-sum price of P4.2
million. The ordinary shares are currently selling at P145 per share while the preference shares are currently
selling at P85 per share.

3. On May 3, the Company reissued 2,000 of the treasury shares at P138 per share.

4. On June 20, the Company issued stock rights to its ordinary shareholders. The stock rights shall entitle the
holder to acquire 1 additional ordinary shares for every 5 stock rights. The stock rights with an exercise price
of P125 shall expire on October 1.

5. On July 30, the Company retired 3,000 of the treasury shares and reverted them to unissued basis.

6. September 30, all but 22,000 share rights were exercised.

7. On December 20, the Company declared P8 cash dividends to ordinary shares and P12 cash dividends to
preference shares. The cash dividends are payable to stockholders as of December 31, payable on January 20
of the following year.

8. Net income for the period was at P1,654,000.

What is the total stockholders’ equity as of December 31, 2016?


a. 24,684,800 c. 24,864,800
b. 24,884,800 d. 24,486,800
____ 34. Which of the following is least likely to be comparable between similar corporations in the
same industry line of business?
a. earnings per share
b. accounts receivable turnover
c. return on total assets before interest and taxes

Page 32 of 76
d. operating cycle
____ 35. One of the major parts of audit planning is pre-planning. Which of the following is not
involved during the pre-planning phase?
a. obtaining an engagement letter
b. selecting staff for the engagement
c. obtaining information about client’s legal obligations
d. deciding whether to accept or continue an audit engagement
____ 36. The following provide the details of a sales and purchases cut-off rendered by your audit staff
in line with your audit of Grace Corporation’s financial statements as of and for the period ended December
31, 2016. The inventories reported per books amounting to P339,900 was as a result of a physical count
conducted on the client’s warehouse on December 30, 2016. All customers are within a 3-5 days delivery
area. Gross profit on sales is at 40%.

The following is a summary of the cut-off made on sales transactions:

December 2016 entries on the sales journal


Invoice No. Invoice date Shipment date Amount Remarks
19817 Dec. 19 Dec. 20 38,250 FOB shipping point
19818 Dec. 21 Dec. 21 40,450 FOB destination (to consignee)
19819 Dec. 27 Dec. 29 45,250 FOB destination (in transit)
19820 Dec. 29 Dec. 31 25,300 FOB shipping point (in transit)

January 2017 entries on the sales journal


Invoice No. Invoice date Shipment date Amount Remarks
19821 Dec. 31 Dec. 31 43,400 FOB shipping point (in transit)
19822 Jan. 2 Jan. 2 40,450 FOB destination
19823 Jan. 4 Jan. 5 45,250 FOB shipping point

The following is a summary of the cut-off made on purchases transactions:

December 2016 entries on the purchase journal


Receiving Receiving Amount Remarks
Report No. Report Date
2813 Dec. 15 18,100 FOB shipping point
2814 Dec. 26 25,250 FOB destination (to consignee)
2815 Dec. 31 14,950 FOB destination
2816 Jan. 2 15,400 FOB destination (in transit)

January 2017 entries on the purchases journal


Receiving Receiving Amount Remarks
Report No. Report Date
2817 Jan. 2 12,750 FOB shipping point (in transit)
2818 Jan. 4 16,250 FOB destination
2819 Jan. 5 20,700 FOB shipping point

The unadjusted balance of the accounts receivable account and accounts payable accounts as of December 31,
2016 were at P395,300 and P210,700, respectively.
What is the adjusted balance of inventories as of December 31, 2016?
a. 367,730
b. 339,800
c. 352,550
d. 337,600
____ 37. Which of the following is most accurate with respect to a CPA’s responsibility in considering
a going concern question on audits?
a. determine that related uncertainties are properly discussed and make no mention in the
audit report
b. issue a report with a “going concern” modification when failure is at least reasonably
probable.
c. based on audit procedures performed, assess whether there is substantial doubt about the
entity’s ability to continue as a going concern
d. perform analytical procedures aimed particularly at assessing whether bankruptcy is
probable

Page 33 of 76
____ 38. Which of the following activities would most likely be performed in the IT department?
a. initiation of changes to existing application
b. correction of transactional errors
c. conversion of information to machine-readable form
d. initiation of changes to master records
____ 39. Which of the following statements is true?
a. Statistical sampling requires quantitative audit procedures whereas nonstatistical sampling
requires judgmental audit procedures.
b. The audit procedures will be the same for either statistical or nonstatistical sampling but
they must be performed differently for each.
c. Audit procedures on the sample item will vary as a result of using either statistical or
nonstatistical sampling.
d. Audit procedures on the sample item will not vary as a result of using either statistical or
nonstatistical sampling.
____ 40. In a computerized information system, automated equipment controls or hardware controls
are designed to
a. correct errors in the computer programs
b. monitor and detect errors in source documents.
c. arrange data in a logical sequential manner for processing purposes
d. detect and control errors arising from use of equipment

Page 34 of 76
Mock Exam: Auditing
Answer Section

MULTIPLE CHOICE

1. C
2. B
3. B
4. C
5. B
6. C
7. B
8. B
9. A
10. D
11. D
12. D
SOL:
Gross sales, accrual
Cash collection from customers 45,780,000
Add: 1,580,000
Sales discounts 1,590,000
Sales returns, excluding refunds 1,120,000
Write-off of receivables 1,900,000
Decrease in advances from customers 4,200,000
Increase in accounts receivables 56,170,000
Total
Less: Recoveries of previous write-off (420,000)
Accrual basis gross sales 55,750,000

Gross purchases, accrual


Cash payments to suppliers of inventory 24,490,000
Add:
Purchase discounts 1290,000
Purchase returns, excluding refunds 1,320,000
Increase in accounts payable 3,780,000
Total 30,880,000
Less: Increase in advances to suppliers (1,512,000)
Accrual basis gross purchases 29,368,000

Cost of sales, accrual


Accrual basis gross purchases 29,368,000
Less:
Purchase discounts 1,290,000
Purchase returns 1,960,000
Net purchases 26,118,000
Less: Increase in inventories 2,690,000
Cost of sales 23,428,000

Bad debts expense


Increase in allowance for bad debt 840,000
Add:
Write off of AR 1,120,000
Less: Recovery of previous written off 420,000
Bad debt expense 1,540,000

Depreciation expense
Increase in equipment account 2,000,000
Purchase of equipment during the year 5,000,000
Cost of equipment sold during the year 3,000,000
Carrying value of equipment sold 1,600,000

Page 35 of 76
Accumulated depreciation of equipment sold
during the year 1,400,000

Increase in accumulated depreciation 1,000,000


Decrease in accumulated depreciation from
sold equipment 1,400,000
Depreciation expense for the year 2,400,000
13. B
14. D
15. C
SOL:
Investment in Miascor Shares (Associate)
Initial cost (6/30/2014) 2,400,000
Share from dividends (P5 x 50,000 shares) (250,000)
Share from net income (2,240,000 x 6/12 x 25%) 280,000
Investment in Associate balance 2,430,000
16. D
17. D
18. D
19. B
20. C
21. D
SOL:
Amortization
Patent (P200,000 / 10 years) 20,000
Computer software (100,000 x 60/120) 50,000
Total amortization 70,000

Impairment loss
Copyright
Carrying value 400,000
Recoverable value/value in use (P8,000/5%) 160,000 240,000
Tradename
Carrying value 350,000
Recoverable value/value in use (P15,000/5%) 300,000 50,000
Goodwill
Carrying value of the CGU including goodwill 3,000,000
Recoverable value/value in use (P200,000 x 14.093) 2,818,789 181,211
Total impairment loss 471,211

Carrying value of goodwill after impairment loss


Goodwill before impairment loss 900,000
Impairment of the CGU entirely attributed to goodwill (181,211)
Carrying value of goodwill after impairment loss 718,789

Carrying value of other intangibles


Patent, 12/31/2014 (P200,000 - P20,000) 180,000
Computer software (P100,000 - P50,000) 50,000
Copyright 160,000
Tradename 300,000
Carrying value of intangibles as of 12/31/2014 690,000
22. C
23. D
24. C
25. D
26. C
27. A
28. D
SOL:
Total current assets:
Cash 114,000

Page 36 of 76
Trading securities 80,000
Receivable, net 160,000
Inventories 180,000
Total current assets 534,000

Non current assets:


PPE:
Land 500,000
Building, net 680,000
Equipment, net 270,000 1,450,000
Investments:
Available for sale securities 270,000
Land held for future use 270,000
Cash surrender value 40,000
Bond sinking fund 250,000 830,000
Franchise 165,000
Goodwill 100,000
Total non current assets 2,545,000

Assets
Total current assets 534,000
Total noncurrent assets 2,545,000
Total assets 3,079,000
29. A
30. B
SOL:
Amortization table, January 2011 to December 2014
Correct interest Nominal interest
(Carrying value x 12%) (Principal x 10%) Amortization Balance
January, 2011 9,279,045
December 2011 1,113,485 1,000,000 113,485 9,392,530
December 2012 1,127,104 1,000,000 127,104 9,519,634
December 2013 1,142,356 1,000,000 142,356 9,661,990
December 2014 1,159,439 1,000,000 159,439 9,821,429

Impairment loss
Loans and receivable, including interest (9,821,429 +
1,000,000) 10,821,429
Present value of future cash flows using the original
effective rate 12%
(1,000,000 x 0.8929) 892,857
(2,000,000 x 0.7972) 1,594,388
(2,500,000 x 0.7118) 1,779,451
(2,500,000 x 0.6355) 1,588,795 5,855,491
Impairment loss 4,965,938

Amortization table, December 2014 onwards


Amortization Principal
(Carrying value x 12%) collection Balance
December, 2014 5,855,491
December 2015 702,659 1,000,000 5,558,150
December 2016 666,978 2,000,000 4,225,128
December 2017 507,015 2,500,000 2,232,143
December 2018 267,857 2,500,000 -
31. D
32. D
SOL:
Capitalized lease asset
Present value of periodic payments (550,000 x 4.1699) 2,293,426
Present value of BPO (250,000 x 0.6209) 155,230
2,488,656

Page 37 of 76
Carrying value of lease asset
Capitalized cost of the asset, December 31, 2013 2,488,656
Accumulated depreciation (2014) (2,488,656/12) (408,109)
Carrying value, December 31, 2014 2,244,601

Amortization table, 12% bonds payable


Date Nominal interest Effective interest Amortization Balance
1/1/2014 3,231,652
6/30/2014 180,000 161,583 (18,417) 3,213,235
12/31/2014 180,000 160,662 (19,338) 3,193,897
Total 322,245

Amortization table, 12% bonds payable


Date Periodic Effective interest Principal Balance
payments
12/31/2013 2,488,656
12/30/2013 550,000 - 550,000 1,898,656
12/31/2014 550,000 189,866 360,134 1,538,522
12/31/2015 550,000 153,852 396,148 1,142,374

Long-term Short-term Interest expense


Note payable 3,000,000 1,500,000 -
Interest
(P6 M x 10% x 8/12) 400,000
(P4.5 M x 10% x 4/12) 500,000
Bonds payable 3,193,897 322,245
Lease Liability 1,142,374 396,148 189,866
Total 7,336,271 1,896,148 1,062,111
33. C
SOL:
Share Share
Ordinary Preference premium - premium - Accumulated Treasury
share share ordinary preference profit share
8,000,000 5,000,000 2,400,000 1,000,000 3,890,000
1 (1,168,000)
2 2,000,000 500,000 1,248,000 452,000
3 (16,000) 292,000
4
5 (300,000) (90,000) (48,000) 438,000
6 1,440,000 360,000
7 (2,187,200)
1,654,000
8 11,140,000 5,500,000 3,918,000 1,452,000 3,292,800 (438,000)

Stockholders’ equity
Ordinary shares 11,140,000
Preference shares 5,500,000
Share premium - ordinary 3,918,000
Share premium - preference 1,452,000
Accumulated profits - unappropriated (3,292,800 - 438,000) 2,854,800
Accumulated profits - appropriated for treasury 438,000
Treasury shares (438,000)
Total stockholders’ equity 24,864,800
34. A
35. C
36. C
SOL:
Accounts Accounts
Inventory receivable payable Net income
Unadjusted balances 339,900 395,300 210,700
December sales entries:
SI 19818 to consignee 24,270 (40,450) (16,180)

Page 38 of 76
SI 19819 in transit FOB Dest. 27,150 (45,250) (18,100)
SI 19820 in transit FOB SP (15,180) (15,180)
January sales entries:
SI 19821 in transit FOB SP (26,040) 43,400 17,360
December purchase entries:
RR 2814 from consignor (25,250) (25,250)
RR 2815 14,950 14,950
RR 2816 in transit FOB Dest. (15,400) 15,400
January purchase entries
RR 2817 in transit FOB SP 12,750 12,750
352,500 353,000 182,800 (1,750)
37. C
38. C
39. D
40. D

Page 39 of 76
FINANCIAL ACCOUNTING AND REPORTING
FIFTH YEAR

____ 1. Alysa Company owns an office building and leases the offices under a variety of rental
agreements involving rent paid in advance monthly or annually. Not all tenants make timely payments of their
rent. The following data were taken from the balance sheets of Alysa Company:

During 2016, Alysa received P800,000 cash from tenants.

What amount of rental revenue should Alysa record for 2016?


a. 908,000
b. 852,000
c. 692,000
d. 748,000
____ 2. The purchase of treasury shares
a. Decreases total assets and decreases total shareholders' equity
b. Has no effect on total assets, total liabilities or total shareholders' equity.
c. Increases one asset and decreases another asset
d. Decreases total assets and increases total shareholders' equity.
____ 3. Sandal Ltd (Sandal) prepares consolidated financial statements. During the financial year
ended 30 June 20X4, Sandal disposed of an investment in a foreign operation. Up to the date of disposal,
Sandal had to translate the financial statement of the foreign operation from another currency for inclusion in
its consolidated financial statements. During prior reporting periods, P14,000 of exchange difference gains net
of tax (pre-tax exchange difference gains P20,000) had been recognized in other comprehensive income in the
consolidated financial statements of Sandal. During the 20X4 reporting period, a P3500 exchange difference
gain net of tax (pre-tax exchange difference gain P5,000) up to the date of disposal of the foreign operation
had been recognized in other comprehensive income.

In accordance with PAS 1 Presentation of Financial Statements, which one of the following statements is
correct in relation to the treatment of the disposal of the foreign operation in the consolidated statement of
profit or loss and other comprehensive income of Sandal for the year ended 30 June 20X4?
a. No reclassification adjustment from other comprehensive income to profit or loss is
necessary on disposal of the foreign operation.
b. Other comprehensive income would include a reclassification adjustment net of tax of
P17,500.
c. Other comprehensive income would include a reclassification adjustment net of tax of
P14,000.
d. Other comprehensive income would include an exchange difference net of tax gain of
P3,500.
____ 4. Are the following statements in relation to the term ‘dilution’ true or false, according to PAS
33 Earnings per share?

I A reduction in earnings per share is an example of dilution


II A reduction in loss per share is an example of dilution

a. True, True
b. True, False
c. False, True
d. False, False
____ 5. The following information is available for the property, plant and equipment of Fry as at 30
September:

2020 2019
P’000 P’000
Carrying amounts 23,400 14,400

The following items were recorded during the year ended 30 September 2020:
I. Depreciation charge of P2·5 million
II. An item of plant, with a carrying amount of P3 million, was sold For P1·8 million.
III. A property was revalued upwards by P2 million

Page 40 of 76
IV Environmental provisions of P4 million relating to property, plant and equipment
were capitalized during the year What amount would be shown in Fry’s statement of
cash flows for purchase of property, plant and equipment for the year ended 30
September 2020?

a. P7.3 million
b. P12.5 million
c. P10.5 million
d. P8.5 million
____ 6. Dave Corporation reports on a calendar-year basis. Its 2015 and 2016 financial statements
contained the following errors:

2015 2016
Over (under) statement of ending inventory (10,000) 4,000
Depreciation understatement 4,000 6,000
Failure to accrue salaries at year-end 8,000 12,000

As a result of the above errors, 2016 income would be


a. overstated by P4,000
b. overstated by P22,000
c. overstated by P16,000
d. overstated by P24,000
____ 7. Miguel Company, had net income for 2020 of P5,000,000. Additional information is as
follows:

Amortization of patents 45,000


Depreciation on plant assets 1,650,000
Long-term debt:
Bond premium amortization 65,000
Interest paid 900,000
Provision for doubtful accounts:
Current receivables 80,000
Long-term nontrade receivables 30,000

What should be the net cash provided by operating activities in the statement of cash flows for the year ended
December 31, 2020, based solely on the above information?
a. 6,820,000
b. 6,870,000
c. 6,740,000
d. 6,840,000
____ 8. Karen and Andrea are currently changing their partnership profit and loss ratios from 75/25 to
60/40. They have created a list of assets that have market and book value differences. One of the assets is a
building with a P300,000 market value and P200,000 book value. Two years after changing the profit and loss
ratios, the building is sold for P380,000. How much of the profit is allocated to Karen?
a. 135,000
b. 123,000
c. 108,000
d. 183,000
____ 9. A competitor has sued an entity for unauthorized use of its patented technology. The amount
that the entity may be required to pay to the competitor if the competitor succeeds in the lawsuit is
determinable with reliability, and according to the legal counsel it is less than probable but more than remote
that an outflow of the resources would be needed to meet the obligation. The entity that was sued should at
year-end
a. Make no provision or disclosure and wait until the lawsuit is finally decided and then
expense the amount paid on settlement
b. Make a disclosure of the possible obligation in the notes to financial statements
c. Set aside as an appropriation an amount based on the best estimate of the possible liability
d. Recognize a provision for this possible obligation
____ 10. Which of the following is an example of managing earnings down?
a. Reducing research and development expenditures
b. Revising the estimated life of equipment from 10 years to 8 years

Page 41 of 76
c. Changing estimated bad debts from 3% to 2.5% of sales
d. Not writing off obsolete inventory
____ 11. Mark Farm purchased 100 beef cattle at an auction for P800,000 on July 1, 2019.
Transportation costs were P8,000. Mark Farm would have to incur the same transportation costs if it had sold
its cattle in the auction. In addition there would be a 2% auctioneer’s fee on the market price of the cattle
payable by the seller. Mark Farm also incurred P4,000 veterinary expenses. On December 31, 2019, the fair
value of the cattle in the most relevant market increases to P880,000. On May 2, 2020, Mark Farm sold 18
cattle at auction for P160,000 and incurred transportation charges of P1,200.

On June 15, 2020, the fair value of the remaining cattle was P662,560 but on the same day, 42 cattle were
slaughtered with total cost of P33,600. The fair value of the carcasses on that day was P386,400 and the
estimated transportation cost to sell the carcasses is P3,360. No other selling costs are expected.

On June 30, 2020, the fair value of the remaining 40 cattle was P358,400. The estimated transportation cost is
P3,200.

What is the net proceeds from the sale of cattle on May 2?


a. 160,000
b. 158,800
c. 155,600
d. 156,800
____ 12. A company issues fully paid shares to 500 employees on 31 July 2020. Shares issued to
employees normally have vesting conditions attached to them and vest over a three-year period, at the end of
which the employees have to be in the company’s employment. These shares have been given to the
employees because of the performance of the company during the year. The shares have a market value of
P2,000,000 on 31 July 2020 and an average fair value for the year of P3,000,000. It is anticipated that in
three-years’ time there will be 400 employees at the company.

What amount would be expensed to profit or loss for the above share issue?
a. 2,000,000
b. 666,667
c. 1,000,000
d. 3,000,000
____ 13. Long-term debt that matures within one year and is to be converted into shares should be
reported:
a. as a current liability
b. in a special section between liabilities and equity
c. as part current and part non-current
d. as non-current if the refinancial agreement is completed by the end of the year
____ 14. Brocken Company has the following data related to an item of inventory:

Inventory, May 1 1,000 units at P4.20


Purchase, May 7 3,500 units at P4.40
Purchase, May 16 700 units at P4.50
Inventory, May 31 1,300 units

The value assigned to cost of goods sold if Brocken uses FIFO is


a. 17,082
b. 16,900
c. 17,069
d. 16,960
____ 15. Carter Corporation acquired two inventory items at a lump-sum cost of P50,000. The
acquisition included 3,000 units of product LF, and P7,000 units of product 1B. LF normally sells for P15 per
unit, and 1B for P5 per unit. If Carter sells 1,000 units of LF, what amount of gross profit should it recognize?
a. 5,625
b. 1,875
c. 11,875
d. 10,000
____ 16. The Conceptual framework for financial reporting defines recognition as the process of
incorporating in the financial statements an item which meets the definition of an element and satisfies certain
criteria.

Page 42 of 76
Which of the following elements should be recognized in the financial statements of an entity in the manner
described?
a. In revenue: the whole of the proceeds from the sale of an item of manufactured plant
which has to be maintained by the seller for three years as part of the sale agreement
b. As a trade receivable: an amount of P10,000 due from a customer which has been sold
(factored) to a finance company with no recourse to the seller
c. In equity: irredeemable preference shares
d. As a non-current liability: a provision for possible hurricane damage to property for a
company located in an area which experiences a high incidence of hurricanes
____ 17. On September 30, 2016, a fire at Stall Company’s only warehouse caused severe damage to
its entire inventory. Based on recent history, Stall has a gross profit of 30% of net sales. The following
information is available from Stall’s records for the nine months ended September 30, 2016:

Inventory at 1/1/2016 600,000


Total purchases received and recorded from January to date of fire 4,000,000
Total freight cost of goods purchased and received 250,000
Total discounts taken on purchases 90,000
Invoice received for goods purchased but still in transit shipped on September
30, 2016, FOB shipping point excluding the freight cost of P5,000 150,000
Invoice purchased but still in transit, Free alongside the carrier (including the
freight and insurance cost from of P15,000) 250,000
Total sales delivered and recorded from January to date of fire 3,700,000
Unrecorded sales invoice for goods delivered 400,000
Total sales returns accounted and recorded to date of fire 100,000
Total sales return recorded but the goods has yet to be received 80,000
Total discounts not taken by customers 60,000

It is the company’s policy to record sales and receivables net of 2% cash discount.

A physical inventory disclosed usable damaged goods which Stall estimates can be sold to a jobber for
P50,000. What is the estimated cost of inventory destroyed by fire?
a. 1,546,900
b. 1,432,900
c. 1,490,900
d. 1,951,900
____ 18. Tibet acquired a new office building on 1 October 2019. Its initial carrying amount consisted
of:

P’000
Land 2,000
Building structure 10,000
Air conditioning system 4,000
16,000

The estimated lives of the building structure and air conditioning system are 25 years and 10 years
respectively. When the air conditioning system is due for replacement, it is estimated that the old system will
be dismantled and sold for P500,000. Depreciation is time apportioned where appropriate.

At what amount will the office building be shown in Tibet’s statement of financial position as at 31 March
2020?
a. 15,625
b. 15,585
c. 15,250
d. 15,600
____ 19. Which of the following is not a legal restriction related to profit distributions by a
corporation?
a. Dividends must be in full agreement with the capital contracts as to preferences and
participation.
b. Profit distributions must be formally approved by the board of directors
c. The amount distributed in any one year can never exceed the net income reported for that
year
d. The amount distributed to owners must be in compliance with the laws governing
corporations

Page 43 of 76
____ 20. Where is debt callable by the creditor reported on the debtor's financial statements?
a. Current liability if it is probable that creditor will call the debt within the year, otherwise a
noncurrent liability
b. Current liability
c. Current liability if the creditor intends to call the debt within the year, otherwise a
noncurrent liability
d. Noncurrent liability
____ 21. At the beginning of Year 1, an entity grants to a senior executive 3,000 share options,
conditional upon the executive’s remaining in the entity’s employ until the end of Year 3. The exercise price
is P40. However, the exercise price drops to P30 if the entity’s earnings increase by at least an average of
10% per year over the three-year period.

On grant date, the entity estimates that the fair value of the share options, with an exercise price of P30, is P15
per option. If the exercise price is P40, the entity estimates that the share options have a fair value of P12 per
option.

During Year 1, the entity’s earnings increased by 12%, and the entity expects that earnings will continue to
increase at this rate over the next two years. The entity therefore expects that the earnings target will be
achieved, and hence the share options will have an exercise price of P30. During Year 2, the entity’s earnings
increased by 13%, and the entity continues to expect that the earnings target will be achieved.

During Year 3, the entity’s earnings increased by only 3%, and therefore the earnings target was not achieved.
The executive completes three year’s service, and therefore satisfies the service condition. Because the
earnings target was not achieved, the 3,000 vested share options have an exercise price of P40.

What is the cumulative compensation expense for Years 1, 2 and 3?


a. 36,000
b. 114,000
c. 40,000
d. 60,000
____ 22. In preparing the 20X4 financial statements of Medal Ltd (Medal), there was a voluntary
change of accounting policy in relation to inventories. The accountant for Medal noted that this change would
not require any adjustment in the financial report for the reporting period ending on 30 June 20X4. However,
the accountant considered that the change in accounting policy would have a material effect on the subsequent
reporting period.

In accordance with PAS 8 Accounting Policies, Changes in Accounting Estimates and Errors, which one of
the following actions should be taken when preparing the financial report for the year ended 30 June 20X4?
a. No information about the accounting policy change needs to be disclosed.
b. Disclosure of the nature and reason for the change and that no adjustments were
recognized in the 20X4 financial statements.
c. Disclosure of the nature and reason for the change and its estimated financial effect in the
subsequent reporting period.
d. Inclusion of a note stating that an accounting policy had been changed but no adjustments
were required in the 20X4 financial statements.
____ 23. Members' shares in cooperatives shall be classified as equity

I If the entity has the unconditional right to refuse redemption of members shares.
II If the redemption of members' shares is unconditionally prohibited by law.

a. II only
b. Neither I nor II
c. Either I or II
d. I only
____ 24. Drumlin Co has P5m of P0.50 nominal value ordinary shares in issue. It recently announced a
1 for 4 rights issue at P6 per share. Its share price on the announcement of the rights issue was P8 per share.

What is the theoretical value of a right per existing share?


a. 0.50
b. 1.50
c. 1.60
d. 0.40

Page 44 of 76
____ 25. Under PAS 8, management shall use its judgment in developing and applying an accounting
policy. In making this judgment, which of the following shall be considered as the least authoritative source?
a. Requirement and guidance in PFRS dealing with similar and related issue.
b. The definitions, recognition criteria and measurement concepts for assets, liabilities,
income and expenses in the Framework.
c. The requirement of a particular PFRS or an Interpretation that specifically applies to a
transaction or a condition.
d. Accounting literature and popular industry practices
____ 26. On 1 October 2013, Xplorer commenced drilling for oil from an undersea oilfield. The
extraction of oil causes damage to the seabed which has a restorative cost (ignore discounting) of P10,000 per
million barrels of oil extracted. Xplorer extracted 250 million barrels of oil in the year ended 30 September
2014.

Xplorer is also required to dismantle the drilling equipment at the end of its five-year licence. This has an
estimated cost of P30 million on 30 September 2018. Xplorer’s cost of capital is 8% per annum and P1 has a
present value of 68 cents in five years’ time.

What is the total provision (extraction plus dismantling) which Xplorer would report in its statement of
financial position as at 30 September 2014 in respect of its oil operations?
a. 34,900,000
b. 4,132,000
c. 24,532,000
d. 22,900,000
____ 27. A deferred tax liability is computed using
a. Either current or expected future tax law, regardless of whether the expected law has been
enacted.
b. The current tax law, regardless of expected or enacted future tax law.
c. Current tax law, unless enacted future tax law is different.
d. Expected future tax law, regardless of whether the expected law has been enacted.
____ 28. Which of the following is a correct statement of one of the capitalization criteria for leases?
a. The lease contains a purchase option
b. The lease transfers ownership of the property to the lessor
c. The minimum lease payments excluding executory costs equal or exceed 90% of the fair
value of the leased property.
d. The lease term is equal to or more than 75% of the estimated economic life of the leased
property
____ 29. Hook Ltd (Hook) purchases an investment property on 1 July 20X0 for P100,000. At 30 June
20X1, Hook determines the fair value of the investment property to be P150,000. At 30 June 20X2, the fair
value of the investment property had fallen to P80,000. Hook’s accounting policy is to carry investment
properties at fair value. Which one of the following journal entries is processed by Hook on 30 June 20X1?
a. Debit: Investment property (50,000); Credit: Asset revaluation reserve (50,000)
b. No entry is required
c. Debit: Investment property (50,000); Credit: Gain on revaluation (profit or loss) (50,000)
d. Debit: Investment property (50,000); Credit: Rental revenue (50,000)
____ 30. The recoverable amount of an asset is its
a. Fair value less cost to sell or value in use whichever is higher
b. Net selling price
c. Fair value less cost to sell or value in use whichever is lower
d. Value in use
____ 31. What is the relationship between present value and the concept of a liability?
a. Present values are used to measure all liabilities.
b. Present values are used to measure certain liabilities.
c. Present values are not used to measure liabilities.
d. Present values are only used to measure noncurrent liabilities.
____ 32. An entity rents out its equipment under an operating lease over a 3-year period to another
entity. The entity has determined that based on the expected pattern of future benefits generated by the
machine, the SYD method of depreciation is the most appropriate. How will the entity recognize income from
the operating lease?
a. Total rental payments by the lessee will be recognized as income over 3 years using SYD.
b. Total rental payments by the lessee will be recognized as income over 3 years using
straight line.

Page 45 of 76
c. Total rental payments by the lessee will be recognized as income immediately.
d. Total rental payments by the lessee will be recognized as income at the end of the 3-year
term.
____ 33. Cesar Company buys and sells securities expecting to earn profits on short-term differences
in price. During 2015, Cesar Company purchased the following trading securities:

Security Cost Fair value Dec. 31, 2015


A 585,000 675,000
B 900,000 486,000
C 1,980,000 2,034,000

Before any adjustments related to these trading securities, Cesar Company had net income of P2,700,000.

What would Cesar’s net income be if the fair value of security B were P855,000.
a. 2,655,000
b. 2,700,000
c. 2,799,000
d. 2,601,000
____ 34. The accountant at Reber Company has determined that income before income taxes
amounted to P6,750 using the FIFO costing assumption. If the income tax rate is 30% and the amount of
income taxes paid would be P225 greater if the average-cost assumption were used, what would be the
amount of income before taxes under the average-cost assumption?
a. 6,975
b. 7,500
c. 6,525
d. 6,090
____ 35. The Jon Company was incorporated on January 1, 2012, with the following authorized
capitalization:

100,000 shares of ordinary shares, no par value, stated value of P40 per share
50,000 shares of 10% cumulative preference, par value P100 per share

During 2012, Jon issued 25,000 ordinary shares for a total of P1,625,000 and 10,000 preference shares at P120
per share. In addition, on December 20, 2012, subscription for 10,000 ordinary shares was taken at a purchase
price of P55 per share. Half of the subscription price was paid for on that date. The balance for the subscribed
shares was paid for on January 2, 2013 and issued on the same date. What should Jon report as total contributed
capital in its December 31, 2012 statement of financial position?
a. 2,825,000
b. 3,100,000
c. 2,400,000
d. 3,375,000
____ 36. Semirara Company has acquired a tract of mineral land for P50 million. Semirara estimates
that the acquired property will yield 150,000 tons of ore with sufficient mineral content to make mining and
processing profitable. It further estimates that 7,500 tons of ore will be mined the first and last year and
15,000 tons every year in between. Assume 11 years of mining operations.The land will have a residual value
of P1,550,000.

Semirara builds necessary structures and sheds on the site at a total cost of P12 million. The company
estimates that these structures can be used for 15 years but, because they must be dismantled if thy are to be
moved, they have no residual value. Semirara does not intend to use the buildings elsewhere.

Semirara installed at the mine was purchased secondhand at a total cost of P3.6 million. The machinery cost
the former owner P9,000,000 and was 50% depreciated when purchased. Semirara estimates that about half of
this machinery will still be useful when the present mineral resources have been exhausted but that
dismantling and removal costs will just about offset its value at that time. The company does not intend to use
the machinery elsewhere. The remaining machinery will last until about one-half the present estimated
mineral ore has been removed and then be worthless. Cost is to be allocated equally between these two classes
of machinery.

What is the estimated depletion charges for the 1st year?


a. 4,945,000
b. 2,422,500
c. 2,522,500

Page 46 of 76
d. 4,845,000
____ 37. Comparability is identified as an enhancing qualitative characteristic in the Conceptual
framework for financial reporting.

Which of the following does not improve comparability?


a. Disclosing discontinued operations in financial statements
b. Restating the financial statements of previous years when there has been a change of
accounting policy
c. Applying an entity’s current accounting policy to a transaction which an entity has not
engaged in before
d. Prohibiting changes of accounting policy unless required by an PFRS or to give more
relevant and reliable information
____ 38. A temporary difference that would result in a deferred tax liability is
a. Interest revenue on municipal bonds.
b. Subscription received in advance
c. Accrual of warranty expense.
d. Excess of tax depreciation over financial accounting depreciation.
____ 39. On January 2, 2015, Julie Company, a medium-sized entity acquired 20% of the outstanding
ordinary shares of Meadow Company for P2,200,000 which included P50,000 transaction cost. This
investment gave Julie the ability to exercise significant influence over Meadow Company. The book value of
the acquired shares was P1,800,000. The excess of cost over book value was attributed to a depreciable asset
which was undervalued on Meadow Company’s balance sheet and which had ten years useful life remaining.
For the year ended December 31, 2015, Meadow Company reported net income of P540,000 and paid cash
dividends of P180,000 on its ordinary.

If Julie Company uses the fair value model to account for its investment and the investment has a fair value of
P2,400,000 on December 31, 2015, what total amount of income and other revenue from investment should
Julie Company disclosed in its December 31, 2015 profit or loss?
a. 286,000
b. 250,000
c. 236,000
d. 200,000
____ 40. Lovely Company leased equipment to Mie Inc. on January 1, 2015. The lease is for an eight-
year period. The first of eight equal annual payments of P900,000 was made on January 1, 2015. Lovely had
purchased the equipment on December 29, 2014, for P4,800,000. The lease is appropriately accounted for as a
sales-type lease by Lovely. Assume that the present value at January 1, 2015, of all rent payments over the
lease term discounted at a 10 percent interest rate was P5,280,000. What amount of interest income should
Lovely record in 2016 as a result of the lease?
a. 490,000
b. 438,000
c. 480,000
d. 391,800

Page 47 of 76
Mock Exam: FAR
Answer Section

MULTIPLE CHOICE

1. A
SOL:
Collection of rentals 800,000
Rental receivable, 2016 124,000
Rental receivable, 2015 (96,000)
Unearned rentals, 2015 320,000
Unearned rentals, 2016 (240,000)
Rental revenue 908,000
2. A
3. B
SOL:
As the foreign operation was sold during the 20X4 reporting period, the total exchange difference gain net of
tax over the life of the foreign operation would be included as a reclassification adjustment in the statement of
profit or loss and other comprehensive income of Sandal Ltd for that financial year. The total exchange
difference gain net of tax to be reclassified from other comprehensive income is P17,500. That is, the prior
period after-tax exchange difference gain of P14,000 plus the current period after-tax exchange difference
gain of P3500.

Paragraph 48 of PAS 21 requires the cumulative amount of the exchange differences relating to that foreign
operation recognized in other comprehensive income to be reclassified from equity to profit or loss on
disposal of the foreign operation
4. B
5. D
SOL:
Cash flow is (in P million):
23.4 - 14.4 b/f + 2.5 dep + 3 disposal - 2 revaluation - 4 non-cash acquisition = 8.5
6. D
SOL:
Effect on profit over (under)
2015 ending inventory, under 10,000
2016 ending inventory, over 4,000
2016 depreciation, under 6,000
2015 accrued salaries, under (8,000)
2016 accrued salaries, under 12,000
Net effect on 2016 profit 24,000
7. C
SOL:
Net income 5,000,000
Amortization of patents 45,000
Depreciation on plant assets 1,650,000
Bond premium amortization (65,000)
Provisions: 6,000
Current receivables 159,000
Long-term nontrade receivables 128,000
Net cash provided by operating activities 6,740,000
8. B
SOL:
(300,000 - 200,000) (.75) + (380,000 - 300,000) (.60) = 123,000
9. B
10. B
11. C
SOL:
Biological asset initial recognition, July 1, 2019
Fair value in most relevant market 800,000
Transportation costs (8,000)
Auctioneer’s fee (800,000 x 2%) (16,000)
Fair value at point of purchase 776,000

Page 48 of 76
Biological asset initial recognition, December 31, 2019
Fair value in most relevant market 880,000
Transportation costs (8,000)
Auctioneer’s fee (880,000 x 2%) (17,600)
Fair value at point of purchase 854,400

Gain as a result of change in value of biological asset, December 31, 2019


Fair value at point of purchase 854,400
Fair value at point of purchase 776,000
Change in fair value - to profit or loss 78,400

Fair value of the inventory (carcasses), June 15


Fair value of remaining carcasses 386,400
Less: Transportation costs 3,360
Fair value of inventory 383,040

Net proceeds from sale of cattle, May 2


Selling price 160,000
Less: Selling expenses
Transportation 1,200
Auctioneer’s fee (160,000 x 2%) 3,200
Net proceeds from sale 155,600
12. A
SOL:
P2,000,000. The issue of fully paid shares is deemed to relate to past service and should be expensed to profit
or loss at 31 July 2020.
13. D
14. D
SOL:
Inventory beginning (1,000 x 4.20) 4,200
Purchase (3,500 x 4.40) 15,400
Purchase (700 x 4.50) 3,150
Total goods available for sale 22,750
Inventory end (700 x P4.50 + (600 x 4.40) 5,790
Total 16,960
15. A
SOL:
LF: 3,000 x P15 = 45,000/80,000 x P50,000 = P28,125
1B: 7,000 x P5 = 35,000 + 45,000 = P80,000
(1,000 x P15) - (28,125 x (1,000/3000) = P5,625
16. C
SOL:
By definition irredeemable preference shares do not have a contractual obligation to be repaid and thus do not
meet the definition of a liability; they are therefore classed as equity.
17. B
SOL:
Total goods available for sale 4,745,000
Estimated cost of sale (P4,100,000 - 180,000/98% x 70%) 2,800,000
Estimated inventory end 1,945,000
NRV of damaged inventory (50,000)
Merchandise in transit (405,000)
Return in transit (80,000/98% x 70%) (57,100)
Estimated cost of inventory damaged 1,432,900
18. A
SOL:
Six months’ depreciation is required on the building structure and air conditioning system.

P’000
Land (not depreciated) 2,000
Building structure (10,000 - (10,000/25 x 6/12)) 9,800
Air conditioning system (4,000 - (3,500/10 x 6/12)) 3,825
15,625
19. C

Page 49 of 76
20. B
21. A
SOL:
Compensation Cumulative
Year Computation expense for period compensation expense
1 (P15 x 3,000 x 1/3) 15,000 15,000
2 (P15 x 3,000 x 2/3) - P15,000 15,000 30,000
3 (P12 x 3,000 x 3/3) - P30,000 6,000 36,000
22. B
SOL:
Refer to paragraph 29 of PAS 8 Accounting Policies, Changes in Accounting Estimates and Errors.

Paragraph 29 of PAS 8 requires disclosure of information about the accounting policy change where the
accounting policy change might have an effect in subsequent reporting periods. It does not require an estimate
of the financial effect in subsequent reporting periods to be disclosed. It does not indicate the nature of, or
reason for, the change in accounting policy
23. C
24. D
SOL:
Value of a right = ((5m x P8 + 1.25m x P6)/6.25 m - P6)/4 shares = P0.4 per share
25. D
26. C
SOL:
Extraction provision at 30 September 2014 is P2.5 million (250 x 10).
Dismantling provision at 1 October 2013 is P20.4 million (30,000 x 0.68).
This will increase by an 8% finance cost by 30 September 2014 = P22,032,000.
Total provision is P24,532,000.
27. C
28. D
29. C
SOL:
PAS 40 Investment Properties requires a gain or loss arising from a change in the fair value of investment
property to be recognized in profit or loss in the period in which the gain or loss arises.
30. A
31. B
32. A
33. C
SOL:
Net income before trading security adjustment 2,700,000
Unrealized loss (P3,465,000 cost - P3,564,000 fair value) 99,000
Net income, as adjusted 2,799,000

Cesar’s net income after making trading security adjustments


Net income before trading security adjustment 2,700,000
Unrealized loss (P3,465,000 cost - P3,195,000 fair value) (270,000)
Net income, as adjusted 2,430,000
34. B
SOL:
Net income before tax (FIFO) 6,750
Tax rate 30%
Income tax payable (FIFO) 2,025
Increase in tax payable (average) 225
Income tax payable (average) 2,250
Divided by tax rate 30%
Net income before tax (average) 7,500
35. D
SOL:
Ordinary shares 1,625,000
Preference shares (10,000 x 120) 1,200,000
Subscribed ordinary shares (10,000 x 550) 550,000
Total contributed capital 3,375,000
36. B
SOL:

Page 50 of 76
Cost Estimated Reserves Depletion/Depreciation
Mineral property 48,450,000 (a) 150,000 323
Building 12,000,000 150,000 80
Machinery (1/2) 1,800,000 150,000 12
Machinery (1/2) 1,800,000 150,000 24 (b)

(a) P50,000,000 - P1,550,000


(b) (P1,800,000/150,000) x 2

Depletion Depreciation
Mineral property (P323 x 7,500) 2,422,500
Building (P80 x 7,500) 600,000
Machinery (1/2) (P12 x 7,500) 90,000
Machinery (1/2) (P24 x 7,500) 180,000
2,422,500 870,000
37. C
SOL:
As it is a new type of transaction, comparability with existing treatments is not relevant.
38. D
39. A
SOL:
Share in the amount of dividend (P180,000 x 20%) 36,000
Unrealized gain (P2,400,000 - P2,150,000) 250,000
Total 286,000
40. D
SOL:
Date Payment Interest (10%) Principal Carrying amount
1/1/2015 - - - 5,280,000
1/1/2015 900,000 - 900,000 4,380,000
a/1/2016 900,000 438,000 462,000 3,918,000
1/1/2017 900,000 391,800

Page 51 of 76
MANAGEMENT ADVISORY SERVICES
FIFTH YEAR

____ 1. Jimmy Company produces two automotive parts, carburetors and air filters. Both products are
made in the same manufacturing facilities but are produced under different processes. To accomplish an
accurate allocation of production costs, the company uses activity based costing. The cost accountant for the
company provided information about the activities used produce the company’s products. The activities were
organized into the following overhead cost categories. The most appropriate cost driver for each category is
also provided:

Category Estimated cost Cost driver Carburetors Air filters


Unit level 60,000 labor hours 900 700
Batch level 22,000 set-ups 20 30
Product level 45,000 storage space 2,000 sq. m 4,000 sq. m
Facility level 100,000 machine hours 7,500 12,500

If carburetors and air filters require the same amount of direct labor, what will be effect if labor hours are used
as the allocation base for product-level costs?
a. air filters and carburetors will be under costed
b. air filters and carburetors will be over costed
c. carburetors will be over costed
d. air filters will be over costed
____ 2. A banking system with a reserve ratio of 20%. A change in reserves of P1,000,000 can
increase its total demand deposit by
a. 1,000,000
b. 5,000,000
c. 800,000
d. 200,000
____ 3. Fonz Inc., expects to sell P20,000 pool cues for P24.00 each. Direct materials costs are P4.00,
direct manufacturing labor is P8.00, and manufacturing overhead is P1.60 per pool cue. The following
expected beginning and desired ending inventory levels apply to 2017:

Beginning Inventory Ending Inventory


Direct materials 24,000 units 24,000 units
Work-in-process inventory 0 units 0 units
Finished goods inventory 2,000 units 2,500 units

How many pool cues need to be produced in 2017?


a. 22,500 cues
b. 19,500 cues
c. 20,500 cues
d. 22,000 cues
____ 4. Julie Corporation has preferred stock with a market value of P107 per share, a face value of
P100 per share, underwriting costs of P5 per share, and annual dividends of P10. Julie’s tax rate is 30%.

What is Julie’s approximate cost of capital for preferred stock?


a. 9.3%
b. 10.5%
c. 6.9%
d. 9.8%
____ 5. Cesar Company has the following information

Breakeven sales 528,000


Variable cost ratio 60%

At a net profit ratio of 8%, what is Cesar’s degree of operating leverage?


a. 5 times
b. 3 times
c. 4 times
d. 2 times

Page 52 of 76
____ 6. Cost volume profit analysis is a key factor in many decisions, including choice of product
lines, pricing of products, marketing strategy, and use of productive facility. A calculation used in a CVP
analysis is the breakeven point. Once the breakeven point has been reached, operating income will increase by
the
a. sales price per unit for each additional unit sold
b. variable costs per unit for each additional unit sold
c. gross margin per unit for each additional unit sold
d. contribution margin per unit for each additional unit sold
____ 7. At the start of the year, Zhoe Company initiated a quality improvement program. The
program was successful in reducing scrap and rework costs. To help assess the impact of the quality
improvement program, the following data were collected for the current year:

Sales 100,000
Scrap 20,000
Rework 25,000
Quality training 10,000
Product warranty 30,000
Product inspection 40,000
Materials inspection 20,000

In a quality cost report prepared by the Zhoe Company, what total amount should be shown as non-
conformance costs?
a. 70,000
b. 75,000
c. 115,000
d. 45,000
____ 8. Jen, Inc., began operations on January 1 of the current year with a P12,000 cash balance.
Forty percent of sales are collected in the month of sale; 60% are collected in the month following sale.
Similarly, 20% of purchases are paid in the month of purchase, and 80% are paid in the month following
purchase. The following data apply to January and February:

January February
Sales 35,000 55,000
Purchases 30,000 40,000
Operating expenses 7,000 9,000

If operating expenses are paid in the month incurred and include monthly depreciation charges of P2,500,
determine the change in Jen’s cash balance during February.
a. P5,000 increase
b. P7,500 increase
c. P2,000 increase
d. P4,500 increase
____ 9. Regan Foundation is a tax-exempt charitable organization. Regan invested P400,000 in a
five-year project at the beginning of year 1. Regan estimates that the annual savings from this project will
amount to P130,000. The P400,000 of assets will be depreciated over their five-year life on the straight-line
basis. On investments of this type, Regan’s required rate of return is 12%. Information on present value
factors is as follows:

12% 14% 16%


PV of P1 for 5 periods 0.57 0.52 0.48
PV of an annuity of P1 for 5 periods 3.60 3.40 3.30

What is the present value of the project?


a. 57,000
b. 68,000
c. 250,000
d. 36,000
____ 10. Cagas Corporation is negotiating with a bank for a P300,000 one-year loan. The loan is
discounted with a 9 percent interest rate and a 20% compensating balance. Suppose that Cagas Corporation
requires the entire amount of P300,000 as net proceeds, how much is the Loan’s required compensating
balance?
a. 60,000
b. 65,934

Page 53 of 76
c. 75,000
d. 84,507
____ 11. Odon is considering an investment in a new machine to replace its existing one. Information
on the existing machine and the replacement machine follow:

Cost of the new machine P40,000


Net annual savings in operating costs 9,000
Salvage value now of the old machine 6,000
Salvage value of the old machine in 8 years 0
Salvage value of the new machine in 8 years 5,000

What is the expected payback period for the new machine?


a. 2.67 years
b. 8.50 years
c. 4.44 years
d. 3.78 years
____ 12. A plant has two categories of overhead: material handling and quality inspection. The costs
expected for these categories for the coming year as follows:

Material handling P100,000


Quality inspection 300,000

The plant currently applies overhead using direct labor, hours and expected actual capacity. This figure is
50,000 direct labor hours.

The plant manager has been asked to submit a bid and has assembled the following data on the proposed job:

Potential Job
Direct materials 3,700
Direct labor (1,000 hours) 7,000
Overhead ?
Number of material moves 10
Number of inspections 5

The manager has been told that many competitors use an activity-based costing (ABC) approach to assign
overhead to jobs. Before submitting a bid, the manager wants to assess effects of this alternative approach.
The manager estimates that the expected number of material moves for all jobs during the year is 1,000; 5,000
quality inspections are expected to be performed.

Assuming the bid price of full manufacturing cost plus 25 percent, what should be the bid under ABC
approach?
a. 18,700
b. 12,000
c. 12,325
d. 15,000
____ 13. Jong Company is considering an investment in a machine that would reduce annual labor
costs by P30,000. The machine has an expected life of 10 years with no salvage value. The machine would be
depreciated according to the straight-line method over its useful life. The company’s marginal tax rate is 30%.

Assume that the company will invest in the machine if it generates an internal rate of return of 16 percent.
What is the maximum amount of the company can pay for the machine and still meet the internal rate of
return criterion?
a. 118,700
b. 187,500
c. 210,000
d. 144,990
____ 14. Morrie Company reports the following balance sheet data:

Current liabilities 280,000


Bonds payable, 16% 120,000
Preferred stock, 14%, P100 par value 200,000
Common stock, P25 par value, 16,800 shares 420,000
Premium on common stock 240,000

Page 54 of 76
Retained earnings 180,000

Income before 40% tax is P160,000 while the common stockholders’ equity in the previous year was
P800,000.

Assume a market price per common share of P35, what is the return on common equity?
a. 8.29%
b. 11.43%
c. 11.71%
d. 8.10%
____ 15. Information on Envy Company’s direct labor cost is as follows:

Standard direct labor rate P12.50


Actual direct labor rate 11.25
Standard direct labor hours 12,000
Direct labor adverse usage variance P14,500

What were the actual hours worked?


a. 15,460
b. 13,160
c. 13,289
d. 14,622
____ 16. When a specified level of safety stock is carried for an item in inventory, the average
inventory level for that item
a. is 50% of the level of the safety stock
b. increases by 50% of the amount of safety stock
c. decreases by the amount pf the safety stock
d. increases by the number of units of the safety stock
____ 17. If a company tends to be more conservative with respect to its working capital policy, then it
would tend to have a(n)
a. increase in the ratio of current assets to current liabilities
b. increase in the normal operating cycle
c. decrease in the ratio of current assets to current liabilities
d. decrease in the normal operating cycle
____ 18. The bailout payback method
a. eliminates the disposal value from the payback calculation
b. measures the risk if a project is terminated
c. incorporates the time value of money
d. equals the recovery period from normal operations
____ 19. Ryan and Raine each operate water-refilling stations. Ryan’s station has an expensive
machine that sucks water from the sea, purifies the water and bottled purified water comes out. His costs are
P5,000 per month plus P2 per bottle of purified water.

Raine’s station collects rainwater and manually filters it using old socks. Her costs are P2,000 per month plus
P4 per bottle of filtered water.

At what number of bottles would the cost per bottle be the same for both?
a. 8,000
b. 6,000
c. 1,500
d. 7,000
____ 20. A company believes that it can sell long-term bonds with a 6% coupon, but a price that gives
a yield-to-maturity of 9%. If such bonds are part of next year’s financing plans, which of the following should
be used for bonds in their after tax (40 percent rate) cost-of-capital calculation?
a. 3.6%
b. 5.4%
c. 4.2%
d. 6.0%
____ 21. Alfonso Company needs 20,000 units of a certain part to use in its production cycle. The
following information is available:

The cost to make the part is as follows:

Page 55 of 76
Direct materials 4
Direct labor 16
Variable overhead 8
Fixed overhead applied 10
Cost to buy the part from Oracle Company 36

If Alfonso buys the part from Cam Sur Company instead of making it, Alfonso could not used the released
facilities in another manufacturing activity. Sixty percent of the fixed overhead applied will continue,
regardless of what decision is made.

In deciding whether to make or buy the part, what is the total relevant cost to make the part?
a. 640,000
b. 720,000
c. 560,000
d. 760,000
____ 22. The annual insurance premium for a factory building would be a:
a. fixed cost, product cost, and direct cost with regard to units of product.
b. fixed cost, period cost, and indirect cost with regard to units of product
c. fixed cost, product cost, indirect cost with regard to units of product
d. variable cost, product cost, direct cost with regards to units of product
____ 23. An organization sells a single product for P40 per unit, which it purchases for P20. The sales
people receive a salary plus a commission of 5% of sales. Last year the organization’s net income (after) taxes
was P100,800. The organization is subject to an income tax rate of 30%. The fixed costs of the organization
are:

Advertising 124,000
Rent 60,000
Salaries 180,000
Other fixed costs 32,000
Total 396,000

The organization is considering changing the compensation plan for sales personnel. If the organization
increases the commission to 10% of sales and reduces salaries by P80,000, what peso sales volume must the
organization have in order to earn the same net income as last year?
a. 1,100,000
b. 1,630,000
c. 1,042,000
d. 1,150,000
____ 24. Joshua will make P500,000 if the fishing season weather is good, P200,000 if the weather is
fair, and would actually lose P50,000 if the weather is poor during the season. If the weather service gives a
40% probability of good weather, a 2% probability of fair weather, and a 35% probability of poor weather,
what is the expected monetary value for Joshua?
a. 232,500
b. 200,000
c. 267,500
d. 500,000
____ 25. For performance evaluation purposes, the variable costs of a service department should be
charged to operating departments using
a. actual variable rate and the budgeted level of activity for the period
b. the budgeted variable rate and the budgeted level of activity for the period.
c. the actual variable rate and the peak-period or long-run average servicing capacity
d. the budgeted variable rate and the actual level of activity for the period.
____ 26. The residual income of a company segment is positive when
a. minimum ROI is lower than the segment’s ROI
b. sales variances are favorable
c. minimum ROI is based on company’s cost of capital
d. costs variances are unfavorable
____ 27. The Company pays a 5% sales commission on each unit sold. If a graph is prepared, with
vertical axis representing “per unit cost” and horizontal axis representing “unit sales” how would a line that
depicts sales commissions be drawn?
a. as a straight diagonal line, sloping upward to the right
b. as a straight diagonal line, sloping downward to the right

Page 56 of 76
c. as a vertical line
d. as a horizontal line
____ 28. The management of Ivory Corporation asks you to prepare an analysis of the gross profit
variance based on their comparative income statements for 2015 and 2016:

2016 2015 Variance


Sales 990,000 800,000 190,000 F
Cost of goods sold 760,000 640,000 120,000 U
Gross profit 230,000 160,000 70,000 F

The only known information given to you is that volume increased from 2015 to 2016 by 10%.

The variance in gross profit due to the change in volume is


a. P70,000 favorable
b. P80,000 favorable
c. P16,000 favorable
d. P64,000 unfavorable
____ 29. Capital budget requirement is P12.5 million. Target debt - equity capital structure is 25%.
Available earnings from operation amounted to P1,500,000. How much of equity capital should be raised?
a. 9,375,000
b. 7,875,000
c. 10,000,000
d. 8,500,000
____ 30. Which of the following example depicts the law of diminishing returns?
a. small electric generating plants are less efficient than large plants
b. An automobile assembly plant has lower per-unit costs at 85% of capacity than at 95% of
capacity.
c. passenger airplanes operate at high costs per passenger when they fly half-empty.
d. a manufacturing company purchases its supplier of materials
____ 31. At the beginning of the year, Tel-aviv initiated a quality improvement program. The program
was successful in reducing scrap and rework costs. To help assess the impact of the quality improvement
program, the following data were collected for the current and preceding years:

Current year Preceding year


Quality training 10,000 15,000
Materials inspection 20,000 35,000
Scrap 200,000 180,000
Rework 250,000 200,000
Product inspection 40,000 60,000
Product warranty 300,000 250,000

Tel-aviv’s internal failure cost must have increased by around


a. 18%
b. 16%
c. 11%
d. 50%
____ 32. An individual receives an income of P3,000 per month and spends P2,500. An increase in
income of P500 per month occurs, an individual spends P2,800. What is the marginal propensity to save?
a. 40%
b. 80%
c. 60%
d. 20%
____ 33. Steve Company’s manufacturing costs for the period just ended were as follows:

Conversion costs 700,000


Other variable manufacturing costs 100,000
Depreciation of factory building and manufacturing equipment 80,000
Other fixed manufacturing overhead 18,000
Variable selling and administrative costs 300,000
Fixed selling and administrative costs 200,000

Page 57 of 76
What amount should be considered product cost for external reporting purposes?
a. 1,100,000
b. 818,000
c. 898,000
d. 1,398,000
____ 34. Bicol Company wants to have an idea of the optimal cash transfer size for 2013 using the
Baumol model. The finance manager has estimated that semiannual cash disbursements for 2013 will total
P27.04 million and the cost per transfer is P400.000. Assuming that interest rate is 16%, how much is the
optimal cash transfer size for 2013?
a. 208,000
b. 367,696
c. 260,000
d. 520,000
____ 35. A company that produce a single product had a net operating income of P85,500 using
variable costing and a net operating income of P90,000 using absorption costing. Total fixed m,manufacturing
overhead was P150,000, and production was 100,000 units. Between the beginning and the end of the year,
the inventory level:
a. decreased by 4,500 units
b. increased by 3,000 units
c. increased by 4,500 units
d. decreased by 3,000 units
____ 36. Cris Company has two service departments and two producing departments. Cost of the
Building and Grounds service department are allocated to other departments on the basis of square footage of
space occupied. The departments in the company and the amount of space occupied. The departments in the
company and the amount of space occupied. The departments in the company and the amount of space
occupied by each department are presented below:

Building and Grounds 2,000 sq. ft.


Cafeteria 8,000 sq. ft.
Producing Department A 20,000 sq. ft.
Producing Department B 30,000 sq. ft.

Budgeted costs in the Building and Grounds department total P34,800 for the year.

Assuming the step method is used and Building and Grounds costs are allocated first, the amount of the Building
and Grounds costs allocated to the Cafeteria would be:
a. 5,568
b. 4,800
c. 4,680
d. 0
____ 37. Why is equity capital generally more expensive than debt financing?
a. investors have a greater demand for equity investments than for debt investments.
b. dividends fluctuate more than interest rates
c. investors expect to be paid more due to exposure to higher risk
d. interest on bonds is a legal obligation
____ 38. The dividends paid in Barreto Company’s outstanding common stock over the past five years
are as follows:

Year Dividend
1 P2.00
2 P2.14
3 P2.29
4 P2.45
5 P2.62

Assuming that the cost of new common stock equity is 12% and there is no flotation cost, what is current market
price of each share?
a. 23.33
b. 52.40
c. 56.00
d. 21.83
____ 39. Steve Corporation has provided the following data from its activity-based costing system:

Page 58 of 76
Activity Cost Pool Total Cost Total Activity
Assembly P710,770 37,000 machine hours
Processing orders P39,690 1,800 orders
Inspection P119,116 1,940 inspection hours

Data concerning one of the company’s products, Product H73N, appear below:

Selling price per unit P125.10


Direct materials cost per unit P34.94
Direct labor cost per unit P49.21
Annual unit production and sales 460
Annual machine hours 510
Annual orders 80
Annual inspections 10

According to the activity-based costing system, the product margin for product H73N is
a. 6,662
b. 7,276
c. 8,426
d. 18,837
____ 40. Silicon Technologies has the following relationships:

Annual sales P1,200,000.00


Current liabilities P 375,000.00
Days sales outstanding (DSO) (365-day a year) 40.00
Inventory turnover ratio 4.80
Current ratio 1.20

The company’s current assets consist of cash, inventories, and accounts receivable. How much cash was
Silicon have on its balance sheet?
a. 125,000
b. 8,333
c. 200,000
d. 68,493

Page 59 of 76
Mock Exam: MAS
Answer Section

MULTIPLE CHOICE

1. C
SOL:
Based on labor hours: 50% vs. 50%
Based on product level: 33.3% vs. 66.7%
2. C
SOL:
Reserves = deposits x reserve ration
1,000,000 = deposits x 20%
3. C
SOL:
20,000 + 2,500 - 2,000 = 20,500 CUES
4. D
SOL:
10 / (107-5) = 9.8%
5. A
SOL:
Degree of operating leverage = Contribution margin / profit)
DOL = 40% divide by 8%
6. D
7. B
SOL:
Non-conformance costs include internal and external failure costs (ex. scrap, rework and warranty)

Scrap 20,000
Rework 25,000
Product warranty 30,000
Total 75,000
8. D
SOL:
Collection of accounts receivable:
February sales P55,000 x 40% 22,000
January sales P35,000 x 60% 21,000 43,000

Payment of purchases:
February purchases P40,000 x 20% 8,000
January purchases P30,000 x 80% 32,000
Cash expenses (P9,000 - P2,500) 6,500 38,500
Increase in cash balance 4,500
9. B
SOL:
Net present value = 130,000 (3.6) - 400,000 = 68,000

The examiner can also ask for the PV factor


PV factor = 400,000 /130,000 = 3.076
10. D
SOL:
[300,000 / (100% - 9% - 20%)] 20% = P84,507
11. D
SOL:
Payback period = (40,000 - 6,000) / 9,000 = 3.78 years
12. D
SOL:
Cost: 3,700 + 7,000 + 100,000 (10/1,000) + 300,000 (5/5,000) = 12,000
Bid price : 12,000 (1.25) = P15,000
13. A
SOL:
[(30,000 x 70%) + (x/10 x 30%)] x 4.833 = x
(21,000 + 0.03X) x 4.833 = x
101,493 + 0.14499x = x

Page 60 of 76
101,493 = 0.855x
x = 118,705
14. A
SOL:
Return in equity = ((60% (160,000) - 14% (200,000))/ ((800,000 + 840,000)/2)
15. B
SOL:
Standard labor cost (12,000 x P12.50) 150,000
Add: Adverse usage variance 14,500
Actual time at standard rate 164,500
Divide by standard labor rate 12.50
Actual hours worked 13,160
16. D
17. A
18. B
19. C
SOL:
5,000 + 2x = 2,000 + 4x
x = 1,500
20. B
SOL:
Cost of debt: Yield rate (100% - tax rate) = 9% (100% - 40%) = 5.4%
21. A
SOL:
(4 + 6 + 8 + 10(40%)) 20,000 = P640,000
22. C
23. D
SOL:
Required peso sales : ((396,000 - 80,000) + (100,800/0.70)) / (40 -(20 +4) = 28,750 units

The examiner can also ask about the breakeven point in unit sales for the organization

396,000 / (40 -2- 2) = 22,000 units


24. C
SOL:
Expected value (100%) = 40% (500,000) + 25% (200,000) + 35% (-50,000) = 267,500
25. D
26. A
SOL:
If ROI > minimum ROI, then residual income increases
If ROI < minimum ROI, the residual income decreases
27. D
28. C
SOL:
SOL:
Sales volume variance P90,000 F
Cost volume variance 64,000 U
Gross profit volume variance P16,000 F

or

2016 units x 2015 gross profit per unit (P160,000 x 110%) P176,000
Less 2015 gross profit 160,000
Gross profit volume variance P 16,000 F
29. D
SOL:
12.5 M (80% equity ratio) - 1.5 M = P8.5 M
Note: If a debt equity ratio is 25%, then debt ratio is 20% (based on 25 / 125)
30. B
31. A
SOL:
(450,000 - 380,000) / 380,000 = 18%

Note: Internal failure costs include, among others, scrap and rework.

Page 61 of 76
32. A
SOL:
Marginal propensity to consume (MPC) = (2,800 - 2,500) /500 = 60%
MPC = MPS = 100%
33. C
SOL:
Conversion costs 700,000
Other variable manufacturing costs 100,000
Depreciation of factory building and manufacturing equipment 80,000
Other fixed manufacturing overhead 18,000
Product cost 898,000
34. D
SOL:
Using Baumol model (pattern: EOQ)
Sq.root: 2(2)27.04 (400) / 0.16 = P520,000
35. B
SOL:
Absorption costing income P90,000
Variable costing income 85,500
Difference in income P 4,500
Divide by fixed overhead cost per unit
P150,000/100,000 1.50
Increase in inventory 3,000
36. B
SOL:
34,800 x (8,000 / 58,000) = P4,800
37. C
38. C
SOL:
12% = [2.62(1.07) / Price] + 7%
Price = P56 (rounded-off)
39. A
SOL:
Sales (P125.10 x 460) 57,546
Less costs:
Materials (P34.94 x 460) 16,072.40
Labor (P49.21 x 460) 22,636.60
Other costs
Assembly (710,770/37,000 x 510) 9,797.10
Processing orders (P39,690/1800 x 80) 1,764
Inspection (P119,116/1,940) 614 50,884.10
Product margin 6,661.90
40. D
SOL:
First, find the amount of the current assets:
Current ratio = Current assets/Current liabilities
Current assets = (Current liabilities) ( Current ratio)
= P375,000(1.2) = P450,000

Next, find the accounts receivables:


DSO = AR/(Sales/365)
AR = DSO (Sales)(1/365)
= (40)(P1,200)(1/365) = P131,506.85

Next, find the inventories:


Inventory turnover = Sales/Inventory
Inventory = Sales/Inventory turnover
= P1,200,000/4.8 = P250,000

Finally, find the amount of cash:


Cash = Current assets - AR - Inventory
= P450,000 - P131,506.85 - P250,000 = P68,493.15 = P68.493

Page 62 of 76
Page 63 of 76
TAXATION
FIFTH YEAR

____ 1. [TRAIN] Carlos, Filipino citizen sold directly to Carrie, shares of stock of DMCI, a domestic
corporation which are classified as capital asset. The selling price was P200,000 at the time when the fair
market value was P250,000 and cost was P100,000.

The capital gains would have been


a. 0
b. 5,000
c. 10,000
d. 15,000
____ 2. Why is the fringe benefits tax collectible from the employer and not from the employee?
a. valuation of benefits is easier at the level of the employer.
b. collection of FBT is ensured because it is withheld at source
c. the employee does not have the means to pay
d. all of the choices are correct
____ 3. May the courts enjoin the collection of revenue taxes?
a. No, because taxes are the lifeblood of the government.
b. No, because taxes are needed to carry out the legitimate objects of the government.
c. No, except if the collection will jeopardize the interest of the taxpayer.
d. No, except if the taxpayer is financially incapable.
____ 4. Which of the following cases is pre-assessment notice required?
a. In a discrepancy between the tax withheld and the amount actually remitted by the
withholding agent
b. When the excise tax due has not been paid
c. Transfer by an exempt person of articles to non-exempt person
d. When the commissioner or his duly authorized representative finds that proper tax should
be assessed.
____ 5. Federico, a Filipino citizen, migrated to the United States some six years ago and got a
permanent resident status or green card. He should pay his Philippine income taxes on
a. the gains derived from the sale in California, U.S.A. of jewelry he purchased in the
Philippines.
b. the proceeds he received from a Philippine insurance company as the sole beneficiary of
life insurance taken by his father who died recently.
c. the gains derived from the sale in the New York Stock Exchange of shares of stock in
PLDT, a Philippine corporation.
d. dividends received from a two year old foreign corporation whose gross income was
derived solely from Philippine sources.
____ 6. A promissory note, which is secured by a real estate mortgage, is subject to the provisions of
the NIRC on documentary stamp tax on loan instruments and documentary stamp tax on real estate mortgages.
The effect of having two documentary stamp tax on the said transaction for the documentary stamp tax is
a. both documentary stamp taxes shall be paid
b. the taxpayer is mandated to pay the higher of the two documentary stamp taxes on the
transaction
c. the taxpayer is allowed to choose between the two transaction on which documentary
stamp tax to pay
d. the earlier dated instrument should be used as basis to determine which documentary
stamp tax is due to be paid.
____ 7. Issues arise as to whether taxes may be collected even without tax measures?
a. yes. because it is an inherent power of the state;
b. yes, because of the life blood theory;
c. no. imposition of taxes without any tax measure amounts to oppression;
d. no. because of absence of the basis for the amount of the tax to be collected;
____ 8. Amaretto, Inc., imported 100 cases of Marula wine from South Africa. The shipment was
assessed duties and value-added taxes of P300,000 which Amaretto, Inc. immediately paid. The Bureau of
Customs did not, however, issue the release papers of the shipment yet since the Food and Drug
Administration (FDA) needed to test the suitability of the wine for human consumption. Is the Bureau of
Customs at fault for refusing to release the shipment just as yet?

Page 64 of 76
a. Yes, because the importation was already terminated as a result of the payment of the
taxes due
b. Yes, the Bureau of Customs is estopped from holding the release of the shipment after
receiving the payment
c. No, if the amount paid as duties and value-added taxes due on the importation was
insufficient
d. No, because the Bureau of Customs has not yet issued the legal permit for withdrawal
pending the FDA’s findings
____ 9. Pierre de Savigny, a Frenchman, arrived in the Philippines on January 1, 2010 and continued
to live and engage in business in the Philippines. He went on a tour of Southeast Asia from August 1 to
November 5, 2010. He returned to the Philippines on November 6, 2010 and stayed until April 15, 2011 when
he returned to France. He earned during his stay in the Philippines a gross income of P3 million from his
investments in the country. For the year 2010, Pierre’s taxable status is that of
a. a non-resident alien not engaged in trade or business in the Philippines.
b. a non-resident alien engaged in trade or business in the Philippines.
c. a resident alien not engaged in trade or business in the Philippines.
d. a resident alien engaged in trade or business in the Philippines.
____ 10. Your client owns a row of apartments. He complains to you that he is being required to pay 4
kinds of taxes on this line of business alone. From the list given by your client, which of the following taxes
has been wrongly imposed on him?
a. Real estate tax on the land and building
b. Value added tax on the gross receipts from rent
c. Community tax based on the assessed value of the apartment house
d. Income tax on income from rent
____ 11. Penafrancia College, Inc., a proprietary educational institution, spent P5 million for the
construction of a new school building. The amount spent for the construction
a. Must be claimed as expense in the year of completion
b. Capitalized and claim annual depreciation over the life of the building
c. Capitalized or expensed outright at the option of the school
d. Capitalized or expensed at the option of the BIR
____ 12. A bought a condominium unit under installment basis, to be used as his office in the practice
of his profession and paying P10,000 monthly. For income tax purposes, the P10,000 monthly payment shall
be
a. Treated as business rental, hence deductible
b. Treated as depreciation expense, hence deductible
c. Treated as capital expenditure, hence not deductible
d. Treated as ordinary business expense
____ 13. A revocable transfer with a consideration received:

Consideration received 200,000


Fair market value of property at the time of transfer 300,000
Fair market value of property at the time of death 250,000

Amount to be included in the gross estate is:


a. 300,000
b. 250,000
c. 100,000
d. 50,000
____ 14. [TRAIN] Mr. Roger died with a receivable from Mr. Novak. Mr. Novak has properties worth
P220,000 and obligations of P320,000. Included in the obligations are P20,000 owed to the Government of
the Republic of the Philippines for unpaid taxes and P60,000 owed to Mr. Roger. The deductible claim
against insolvent persons amount to:
a. 60,000
b. 41,250
c. 20,000
d. 40,000
____ 15. Mr. William died on June 30, 2017, leaving, among others, the following charges and
obligations: Real property tax for the calendar year 2017 - P20,000; On an interest-bearing promissory note
(notarized): face value of the note - P10,000; accrued interest on the note at the time of death - P600; and
interest to accrue on the note from the date of death to the date of maturity - P400. The deduction from the
gross estate is:
a. 20,600

Page 65 of 76
b. 30,600
c. 31,000
d. 21,000
____ 16. A decedent died single, leaving a family home which consists of a piece of a piece of land
that he inherited 3-1/2 years ago (with value at that time of P600,000) with a fair market value of P800,000 at
the time of his death, and a house thereon which he built at a cost of P650,000, and a fair market value at the
time of his death of P450,000. Other properties in his gross estate have a fair market value of P550,000.
Unpaid obligations at the time of his death amounted to P300,000.

The vanishing deduction is equal to:


a. 300,000
b. 500,000
c. 40,000
d. 225,000
____ 17. S, a Filipino decedent, owns a property valued at P2 million at the time of his death. The said
property was sold during his lifetime to V for P1,300,000 when its real value was P1,700,000. It was agreed
by both parties that the delivery and payment will take place after S’ death. For purposes of Philippine estate
tax, the amount to be included in the gross estate would be
a. 2,000,000
b. 1,700,000
c. 700,000
d. 400,000
____ 18. C, VAT-registered person, imported machines to be used in the Philippines as follows:

Machines Purchase price Purpose


1 100,000 Personal use
2 200,000 Business use
3 300,000 For sale

The importations were subjected to 50% excise tax based on purchase price. Machine 3 was sold for P1
million (net of VAT). How much is the VAT paid on importation?
a. 108,000
b. 90,000
c. 54,000
d. 36,000
____ 19. A domestic corporation has the following data for 2014:

Excess MCIT 2013 10,000


Q1 Q2
Income, net of 1% witholding tax 495,000 792,000
Deductions 480,000 700,000

How much is the income tax still due and payable in the second quarter?
a. 4,000
b. 6,000
c. 9,000
d. 13,000
____ 20. The records of a closely held domestic corporation show the following data for 2014:

Gross income 1,500,000


Business expenses 600,000
Gain on sale of business asset 60,000
Interest on deposits with Metrobank, net of tax 5,000
Sale of shares of stocks, not listed and traded
Selling price 150,000
Cost 115,000
Dividends from Jane Corporation, domestic 35,000
Dividends paid during the year 120,000
Reserved for building acquisition 300,000

In 2013, the corporation suffered an operating loss of P130,000. This amount was carried forward and
claimed as deduction from gross income in 2014. The income tax due in 2014 is

Page 66 of 76
a. 234,375
b. 249,000
c. 273,937
d. 288,000
____ 21. Aaron, single, had the following from January 1 to June 30, 2016:

Compensation income (net of payroll deductions) 180,000


Deductions made by the employer
SSS housing loan 24,000
SSS premiums contributions 3,600
Philhealth contributions 2,400
Pag-ibig contributions 1,800
Union dues 2,200
Premiums payments on
Life insurance policy 3,000
Health insurance policy 2,400

The taxable compensation income is


a. 154,000
b. 157,000
c. 156,200
d. 133,000
____ 22. A, married on January 1, 2018. She left an estate with a fair market value of P4 million.
During the year, the estate had a gross income of P800,000 and related expenses of P300,000. The
administrator gave P200,000 to B and another P200,000 to C, A’s beneficiaries, 50% of the amounts given to
B and C came from the income of the estate and the other 50% came from the estate. The net taxable income
of the estate in 2018 is
a. 50,000
b. 80,000
c. 250,000
d. 280,000
____ 23. XYZ Corporation, a domestic corporation had the following data during the calendar year
2017:

Gross income 1,000,000


Business connected expenses 400,000
Dividends from: 100,000
Domestic corporation
Foreign corporation, 90% of the gross income was derived from the Philippines 100,000
Foreign corporation, 60% of the gross income was derived from the Philippines 80,000
Foreign corporation, 30% of the gross income was derived from the Philippines 40,000

The taxable income is?


a. 920,000
b. 820,000
c. 748,000
d. 750,000
____ 24. *
Sales (VAT inclusive) 985,600
Purchases of goods sold:
From VAT taxpayers (VAT inclusive) 224,000
From non-VAT taxpayers 33,600
Purchases of services (VAT exclusive)
From VAT taxpayers 50,000
From non-VAT taxpayers 11,200
Payments for utilities (gas and water) 18,360
Salaries of employees 80,000
Operating expenses 100,000

How much is the VAT payable?


a. 74,400
b. 76,243
c. 84,192
d. 85,392

Page 67 of 76
____ 25. Gardo, a citizen of the Philippines and resident of Baguio City died intestate on July 4, 2017.
Among his gross estate is a property inherited from his deceased father who died June 10, 2014. What
percentage of deduction will be used in computing the amount of vanishing deduction?
a. 21%
b. 40%
c. 60%
d. 80%
____ 26. A VAT-registered contractor performed services for his customer in 2016 and billed him
P11,200,000, broken down as follows: P10 million on cost of services plus P1.2 million, 12% VAT. Of the
contract price of P10 million, only P8 million plus VAT thereon was received from the customer in 2016, and
the balance of P2 million plus VAT was received by the contractor in 2017. How much is the taxable gross
receipts of the contractor for 2016 for VAT purposes?
a. P10 million, the total cost of services performed in 2016.
b. P8 million, the amount received from the customer in 2016
c. P8 million plus VAT received from the customer in 2016
d. P11.2 million, the total cost of services performed plus 12% VAT.
____ 27. Juan, student activist, wants to impugn the validity of a tax on text messages. Aside from
claiming that the law adversely affects him since he sends messages by text, what may he allege that would
strengthen his claim to the right to file a taxpayers’ suit?
a. That he is entitled to the return of the taxes collected from him in case the court nullifies
the tax measure.
b. That tax money is being extracted and spent in violation of the constitutionally guaranteed
right to freedom of communication.
c. That he is filing the case in behalf of a substantial number of taxpayers.
d. That text messages are an important part of the lives of the people he represents.
____ 28. * [TRAIN] Kevin, not a dealer in securities, had the following transactions in ordinary shares
of Katrina Co. (a domestic corporation) for 2018 taxable year:

Jan 12 Purchase of 100 shares 100,000


June 20 Sold the shares purchased on Jan. 12 80,000
June 30 Purchased 70 shares 50,000
Oct. 15 Sold the shares purchased on June 30 75,000

How much was the capital gain on sale of shares on October 15?
a. 25,000
b. 3,750
c. 5,500
d. 0
____ 29. If a VAT-registered person issues a VAT invoice or VAT official receipt for a VAT exempt
transaction, but fails to display prominently on the invoice or receipt the words “VAT-exempt sale”, the
transaction shall
a. still be exempt from VAT
b. become taxable and the issuer shall be liable to pay the VAT thereon
c. be effectively subject to zero percent
d. be considered erroneous transaction and must be disregarded
____ 30. International carriers doing business in the Philippines is subject to the
a. 3% percentage tax on gross receipts derived from the transport of cargo and/or mail from
the Philippines to another country.
b. 3% percentage tax on gross receipts derived from the transport of cargo, mail and
passenger from the Philippines to another country.
c. 12% VAT on their gross receipts derived from the transport of cargo from the Philippines
to another country
d. 0% VAT on their gross receipts derived from the transport of cargo and passenger from
the Philippines to another country.
____ 31. The administrator of the estate of Rommel is in a quandary as to which forgiveness of debt
will he include in the gross estate of a decedent. Which of the following will you tell him to include in the
gross estate of his decedent-client?
a. forgiveness of debt which followed after the debtor rendered services in favor of the
creditor-decedent.
b. forgiveness of debt done during the lifetime of the decedent, the debtor did not render
services in favor of the creditor-decedent.
c. forgiveness of debt by a corporation in favor of a stockholder-decedent.

Page 68 of 76
d. forgiveness of debt provided in the will and testament of a decedent in favor of a debtor
who did not render services in favor of the creditor-decedent.
____ 32. A prospective client approaches you to inquire from you whether or not he will be subject to
VAT. He claims that his sale of fresh fruits and fish for the last twelve months exceed the VAT threshold. His
friend who is not an accountant tells him that he will be subject to VAT and has to register under the VAT
system. What advice will you give him?
a. He does not have to register under the VAT system because his transactions are VAT-
exempt.
b. He has to register because his sales have exceeded the VAT threshold.
c. He may optionally register his VAT-exempt sales so he can avail of input tax credit.
d. He does not have to register because the BIR cannot monitor his sales anyway.
____ 33. Which of the following individual income taxpayer is not a resident alien.
a. a person who is not a citizen of the Philippines but resides therein
b. an alien actually present in the Philippines who is not a mere transient
c. an alien who comes to the Philippines for a purpose of such nature that an extended stay is
necessary for its accomplishment, and who makes the Philippines as his temporary home.
d. an alien who comes to the Philippines for a definite purpose, which in its nature may be
promptly accomplished.
____ 34. [TRAIN] The provisions of the family code require the transfer of the share of the guilty
spouse of his/her share in the community property in favor of the innocent spouse, in effect causing a transfer
of the property. The said transfer is
a. subject to Philippine donor’s tax due to the donative intent
b. not subject to the Philippine donor’s tax due to lack of donative intent
c. not subject to the Philippine donor’s tax due to a provision of the Family Code
d. subject to Philippine donors tax because there was a transfer of property for inadequate
consideration
____ 35. Martin donates the following items prior to his death:

House and lot, to his son, mortis causa 1,000,000


Diamond ring to his wife 500,000
Portion of land to the National Government 300,000
Car to his son on account of current birthday 800,000

How much is the gross amount of donations subject to donor’s and estate tax?
a. 800,000; 1,000,000
b. 1,000,000; 0
c. 1,100,000; 800,000
d. 1,600,000; 500,000
____ 36. [TRAIN] Bert and Bernadette decided to donate some of their property to their children
during their lifetime. They asked you what is the most effective and tax efficient way of implementing the
donation to their children over a period of two years, the total value of the property is P5 million to their only
child:
a. Treat the donation as being made by the spouses collectively.
b. Treat the donation as being made by the spouses as being made by the spouses
independently from each other
c. Split the donations over a period of two year made by each parent independently
d. Donate to the child using the exemption of “in contemplation of marriage.”
____ 37. Estate tax should be imposed upon property transferred during life when the transferor
reserves significant powers over the possession or enjoyment of the property. Significant powers are reserved
when the transferee is, by reason of the reserved power, incapable of freely enjoying or disposing of the
interest in the transferred property until the transferor’s death. These transfers exclude:
a. value of trust property, part of the income and principal will be paid to the life beneficiary
for comfort, support and happiness.
b. property under an appointment where the decedent has a power exercisable in favor of his
creditors or creditors of his estate
c. property conditioned upon the transferor’s survivorship.
d. property passing under ascertainable standard relating to the health, education, support, or
maintenance of the decedent.
____ 38. Artny Company, a Canadian corporation, distributed dividends to its Filipinas Inc, a
Philippine company and a stockholder. The dividends are.
a. are not taxable
b. are taxable at 30% or at 15% if the corporation is subject to a tax sparing credit

Page 69 of 76
c. are taxable at 30%
d. are taxable at 25%
____ 39. [TRAIN] Connie bought a parcel of residential land for P1,000,000 sometime in 2005. He
sold the same to Canyon for P10 million on October 15, 2018. The transaction is subject to 6% capital gains
tax.
a. true
b. false, if Rey is engaged in the real estate business
c. false, it is subject to VAT if the sale is in the regular course of trade or business
d. no correct answer given
____ 40. Shirley was allowed to deduct P200,000 bad debts written off in 2016, where she had a net
income before bad debts of P180,000 and a net loss of P20,000. In 2017, she was able to recover the bad
debts written off in full. The bad debts recovery is
a. taxable to the extent of P180,000
b. taxable to the extent of P20,000
c. taxable to the extent of P200,000
d. not taxable

Page 70 of 76
Mock Exam: Taxation
Answer Section

MULTIPLE CHOICE

1. D
SOL:
The capital gain is the selling price of P200,000 less the cost of P100,000, or P100,000. The capital
gain tax is:

If the sale was before the TRAIN LAW, at 10% of the gain, is P10,000.

If the sale was when the TRAIN LAW applies already, at 15% of the gain, is P15,000
2. B
SOL:
There is wisdom in the law by providing for the computation of the fringe benefit by the employer.
And there is a statutory formula that the employee may not be able to handle. There shall not be an
understatement in the amount of the benefit by the employer because the employer knows that he will
be answerable for an under-declaration – all of the facts on the benefits being known to him.

(valuation of benefits is easier at the level of the employer, may also be an acceptable answer.)

This rule applies under the old law and under the TRAIN Law.
3. C
SOL:
A tax should not jeopardize the interest of the taxpayer. A tax should not be confiscatory.
4. D
SOL:
Precisely, pre-assessment notice is required when it is found that the proper tax has to be assessed.
5. C
SOL:
He is a non-resident citizen and he is subject to tax on income from the Philippines. That would be
gain derived from a sale of shares of stock of a domestic corporation.
6. B
SOL:
A documentary stamp tax on the promissory note, and a documentary stamp tax on the contract of
mortgage - which must the taxpayer pay? Preliminarily it must be stated, that failure to pay the stamp
tax on a document does not make the agreement in the document void. Failure to pay the documentary
stamp tax only makes the document unenforceable and not acceptable as evidence in court until the
documentary stamp is paid. Which obligation is sought to be enforced? The creditor may be inclined
on the real estate mortgage, because there is more security on an indebtedness secured by a mortgage
on real property.
7. D
SOL:
While taxation is an inherent power of the state, a crucial question is: How much will be collected?
How much, what time to collect, how will the tax be collected, and how will collection be enforced?
While the state can impose a tax, the tax measure to answer these questions.
8. D
SOL:
Since the articles imported were for human consumption, there is a need for clearance from the Food
and Drug Administration. These are administrative matters that must be complied with under the joint
responsibility of the Food and Drug Administration and the Bureau of Customs. The Bureau of
Customs was not at fault. It was only doing its duty.
9. B
SOL:

Page 71 of 76
The stay outside the Philippines was only eight months in 2010 and six months in 2011. There was no
continuous stay for more than twelve months in 2010 as at the end of 2010 nor for more than twelve
months in 2011 as at the end of 2011. He is a non-resident alien for 2010 and a non-resident alien for
2011, but engaged in business in the Philippines.
10. C
SOL:
The four taxes have different subject matters. The real estate tax is based on value of property in a
certain city, the value-added tax is a tax on the business transaction involving the property, and the
income tax is a tax on the income from the property. The three taxes have different subject matters.
The community tax (used to be called residence tax) is based on residence over a certain period of
time in a locality, even as a measure of the amount is gross receipts or income. The community tax is
not a wrongful tax on business.
11. C
SOL:
The law grants a proprietary educational institution the option either to treat as expense, or to
capitalize, an expenditure to construct a school building - an expenditure towards pursuing its strictly
educational purposes.
12. C
SOL:
The P10,000 periodic payment is not a periodic expense. It is a periodic payment on a purchase price
of an asset. The value at which the property was purchased is a capital expenditure, and should be so
reflected in the Statement of Financial Position.
13. D
SOL:
The rule in the law: The value to include in the gross estate is the excess of fair market value at the
time of death over the consideration received. (P250,000 less P200,000 equals P50,000.)
14. C
SOL:
Value of properties of Mr. Novak P220,000
Obligations of Mr. Novak 320,000
Less: Obligations to preferred creditors 240,000 80,000
Obligations of Mr. Novak that will not be paid - Claim
against insolvent 20,000
15. D
SOL:
Real property (accrued at the beginning of the year) 20,000
Interest on the note (P600 + P400) 1,000
21,000
16. A
SOL:
Initial value to take P600,000
Less: P60,000/1,800,000 x 300,000 100,000
Basis of vanishing deduction 500,000
Vanishing deduction at 60% P300,000
17. A
SOL:
Since delivery and payment is to take place at the time of death, there was no consummated sale
during the lifetime of S. A sale is consummated only when a delivery is made, even if payment is not
yet made. The property was owned by Mr. S up to the time of his death. The value to be included in
his gross estate is P2,000,000.
18. A
SOL:
Machine 1 (P150,000 x 12%) P18,000
Machine 2 (P300,000 x 12%) 36,000
Machine 3 (P450,000 x 12%) 54,000
Total P108,000
19. B
SOL:
Page 72 of 76
Q-1 Q-2
Gross income (P495,000/.99) P500,000
Gross income (P792,000/.99) P800,000
Deductions 480,000 700,000
Taxable income 20,000 100,000
Normal tax at 30% 6,000 30,000
MCIT at 2% of P500,000 10,000
MCIT at 2% P800,000 16,000
Whichever is higher 10,000 30,000
Less: Carry-over from 2013 (10,000)
Carry-over from 1st Q (4,000)
Paid, 1st Q (10,000)
Due P10,000 P6,000
20. B
SOL:
Gross income P1,500,000
Less: Business expenses 600,000
Net income from business P900,000
Add: Gain on sale of business asset 60,000
Total 960,000
Less: Net operating loss carry-forward (130,000)
Taxable income P830,000
Income tax at 30% P249,000
21. C
SOL:
Compensation income P180,000
Add: Mandatory payroll deductions:
SSS premiums 3,600
Philhealth contributions 2,400
Pagibig contributions 1,800 7,800
Total P187,800
Less:
SSS housing loan 24,000
Labor union dues 2,200
Premium payments 5,400 31,600
P156,200
22. C
SOL:
Gross income P800,000
Less: Related expenses 300,000
Net income 500,000
Less: Income distribution:
To A 100,000
To B 100,000 200,000
Net taxable income P300,000
23. B
SOL:
Gross income P1,000,000
Less: Business expenses 400,000
Net income P600,000
Dividend from:
Foreign corporation with 90% Philippine gross income 100,000
Foreign corporation with 60% Philippine gross income 80,000
Foreign corporation with 30% Philippine gross income 40,000
Total P820,000
24. B
SOL:

Page 73 of 76
When an amount given has a tax component (VAT inclusive), the VAT component is determined by
multiplying the amount by the factor of 12/112.)

Output taxes (P985,600 x 12/112) P105,600


Less: Input taxes:
On purchases of goods from VAT taxpayers (P224,000 x 12/112) (24,000)
On purchases of supplies from VAT taxpayers (P50,000 x 12/112) ( 5,357)
Value-added tax payable P76,243

How much was the net income?


Sales (P985,000 less P105,000) P880,000
Less: Purchases of goods
From VAT suppliers (P224,000 less P24,000) 200,000
From non-VAT supplier of goods 33,600
From non-VAT supplier of goods
From VAT suppliers (P50,000 less P5,357) 44,643
Payment of utilities 18,360
Salaries of employees 80,000
Operating expenses 100,000 476,603
Net income P403,379
25. B
SOL:
The time interval between deaths is more than three but not more than four years.
26. B
SOL:
Value-added tax on sale of services is on the gross receipts, regardless of he amount of billing by the
service provider. That will be P8,000,000.
27. B
SOL:
There is freedom of speech, a guaranteed constitutional right to freedom of communication. If the
communicator does not infringe on any right, or privacy of any person, or his communication is not
false, he cannot be prevented from communicating with others.
28. B
SOL:
Selling price P75,000
Less: Cost 50,000
Capital gain 25,000
Capital gain tax at 15% P3,750
29. B
SOL:
A VAT-registered person must issue invoices or receipts which show clearly how much is the VAT. If
the invoice does not show clearly the VAT, or the VAT written there is a wrong amount, the VAT on
the transaction will be computed by multiplying the total by the factor of 12/112. If the invoice or
receipt does not indicate a VAT-exemption, it will be a sale subject to VAT.
30. A
SOL:
The law clearly provides for the tax in (a).
31. D
SOL:
A debt due a person is property of that person. When a person died with that property, he can give that
property by way of succession. Since the indebtedness was forgiven without a consideration (no
services rendered in return), that is giving away of property owned by the creditor, in a will. Property
under the circumstances must be included in the gross estate.
32. A
SOL:
The sale of agricultural and marine products in its original state by the producer of the land or by the
fisherman is exempt from the value-added tax by express provision of law.
33. D
Page 74 of 76
SOL:
A rule applied by regulation and by old consistent practice.
34. B
SOL:
In a donation there must be an intent to give on the part of the benefactor and an intent to receive on
the part of the beneficiary. If either party under the law is disqualified to make or receive a donation,
there can be no donation on which to impose the donor’s tax (or the estate tax). The spouse had no
capacity to receive the property, hence there is no donation.

This rule applies under the old law and under the TRAIN Law.
35. A
SOL:
Subject to the donor’s tax is the donation to the child on account of current birthday of P800,000 and
subject to the estate tax on the donation mortis causa of P1,000,000.
36. C
SOL:
When a donation was made by husband and wife our of joint property, one-half shall be considered by
the husband and one-half shall be considered as made by the wife – The husband: P1,250,000 in year
1, P1,250,000 in year 2, the wife: P1,250,000 in year 1 and P1,250,000 in Year 2.

For husband or wife:

Old law, graduated tax for each year


Year 1 On P500,000 P15,000
700,000 at 8% 56,000
P71,000

Year 2 On P500,000 P15,000


700,000 at 8% 56,000
P71,000

TRAIN, standard annual tax rate:


Year 1 On P 250,000 Exempt
1,000,000 at 6% 60,000
P60,000

Year 2 On P 250,000 Exempt


1,000,000 at 6% 600,000
P60,000
37. A
SOL:
Where part of the income and principal will be paid to the life beneficiary for comfort, support and
happiness, the trustor did not retain control over the property equivalent to retaining absolute
ownership. If there was no absolute ownership on the property as at the time of death, the transferor
upon death did not make a transfer that will be subject to estate tax when he dies.

This rule applies under the old law and under the TRAIN Law.
38. C
SOL:
The receiving corporation, Filipinas Incorporated, is a domestic corporation. It received dividends,
assumed cash dividend, from a foreign corporation. This is a case of a domestic corporation receiving
dividend from a foreign corporation. Dividend received by a domestic corporation from a resident
corporation is income tax from outside the Philippines. The tax on this is 30%. (The 10% final tax
does not apply because the paying corporation is not a domestic corporation.)

This rule applies under the old law and under the TRAIN Law.
39. A
SOL:

Page 75 of 76
This is a sale of real property held as capital asset. The capital gain tax is 6% of the selling price,
whether under the old law or under the TRAIN Law.
40. A
SOL:
Under the “tax benefit rule” income from the recovery of bad debt recovery is to correct the prior
year’s error and to recognize income at the correct amount. The reported net loss came from a
computation:

Gross income before write off P180,000


Less: Bad debts written off 200,000
Net loss 20,000

whereas the correct computation should have been:

Gross income before write off P180,000


Less: True bad debt 0
Correct taxable income P180,000

The error in reporting a net loss of P20,000 must be corrected by reporting a net income of P180,000.

Page 76 of 76

Potrebbero piacerti anche